Medical assessment and management of the surgical patient Flashcards

1
Q
Which of the following infections has a DNA virus as its etiologic agent?
A. Mononucleosis
B. Scrofula
C. AIDS
D. Herpangina
A

<p>Answer: A
Rationale:
It is estimated that 79% of Mononucleosis from an Epstein - Barr virus (EBV) infection; 21 % from a cytomegalovirus (CMV) infection. Both are DNA viruses.
Scrofula (tuberculosis cervical lymphadenitis) is a secondary infection of the cervical lymph node chain, associated with active pulmonary tuberculosis (Mycobacterium tuberculosis) and results in a draining lesion. AIDS is transmitted by Human immunodeficiency virus (HIV) and is a RNA retrovirus. Patients present with opportunistic infections. Herpangina is a Coxsackie virus infection, and is characterized by vesicular eruptions on the fauces and palate. Coxsackie viruses are RNA viruses.
Reference:
Bergman, SA In Topazian, RG, Goldberg, MH, Hupp, JR, editors: Oral and Maxillofacial Infections, ed 4, Philadelphia, 2002, W B Saunders Co., pp 243-278.
McKenna, SJ In Topazian, RG, Goldberg, MH, Hupp, JR, editors: Oral and Maxillofacial Infections, ed 4, Philadelphia, 2002, W B Saunders Co., pp 456-467.
Hupp, JR In Topazian, RG, Goldberg, MH, Hupp, JR, editors: Oral and Maxillofacial Infections, ed 4, Philadelphia, 2002, W B Saunders Co., pp 112-125.
Rubinovitch et al in Cohen & Powderly: Infectious Diseases, 2nd ed., Copyright © 2004 Elsevier, p 200.</p>

How well did you know this?
1
Not at all
2
3
4
5
Perfectly
2
Q
A patient who has had a splenectomy within the last 6 months is most susceptible to which of the following infectious agents?
A. Treponema pallidum
B. Haemophilus influenzae
C. Pneumocystis carinii
D. Mycobacterium tuberculosis
A

<p>Answer: B
Rationale:
Patients who have had a splenectomy are most commonly susceptible to infection by encapsulated bacteria. Haemophilus influenzae is the only bacteria listed which is encapsulated. Following splenectomy, it is recommended that patients be administered the Pneumnovax vaccine to provide active immunity to the encapsulated pneumococcus bacteria.
Reference:
McKenna, SJ In Topazian, RG, Goldberg, MH, Hupp, JR, editors: Oral and Maxillofacial Infections, ed 4, Philadelphia, 2002, W B Saunders Co., pp 456-467
CDC, Morbidity & Mortality Weekly Report: Prevention of Pneumococcal Disease Recommendations of the Advisory Committee on Immunization Practices (ACIP) April 4, 1997 / Vol. 46 / No. RR-8</p>

How well did you know this?
1
Not at all
2
3
4
5
Perfectly
3
Q

<p>Which drug or drug class should be avoided if possible in patients with hypertrophic cardiomyopathy?
A. β- blockers
B. Angiotensin converting enzyme inhibitors
C. Calcium channel blocker
D. Diuretics</p>

A

<p>Answer: D
Rationale:
Dehydration in patients with this condition acts to increase the outflow tract pressure gradients from the heart and generate an increase in symptoms. This can be exacerbated as well with strenuous activity and result in sudden death. Dehydration and the use of diuretics should be avoided if possible so as not to alter this gradient. Digitalis, nitrates, vasodilators and β ñ adrenergic agonists are also to be avoided.
Beta-blockers are the mainstay of medical therapy for hypertrophic cardiomyopathy. Angina, dyspnea, and presyncope may all be improved with beta-blockers. Calcium channel blockers are an alternative therapy to beta blockers. ACE inhibitors are not typically used in the treatment of hypertrophic cardiomyopathy.
Reference:
Kasper, D Braunwald, E et al Harrison's Principles of Internal Medicine 16th ed., McGraw- Hill, New York, 2005 p1411.
Zipes: Braunwald's Heart Disease: A Textbook of Cardiovascular Medicine, 7th ed., 2005, Elsevier, Chapter 59, Hypertrophic Cardiomyopathy.</p>

How well did you know this?
1
Not at all
2
3
4
5
Perfectly
4
Q
<p>Which type of cardiomyopathy is most often associated with high outflow tract pressures?
A. Restrictive
B. Dilated
C. Idiopathic
D. Hypertrophic</p>
A

<p>Answer: D
Rationale:
Patients with hypertrophic cardiomyopathy are most frequently associated with having increased left ventricular outflow tract obstruction. This occurs in approximately 25% of these patients. It is usually related to narrowing of the subaortic area as sequelae of the apposition of the mitral valve leaflet in juxtaposition to the enlarged interventricular septum. Hypertrophic cardiomyopathy is an autosomal dominant inherited disease at least 50% of the time. There are sporadic forms of the disease due to spontaneous mutations.
Dilated cardiomyopathy is characterized by cardiac enlargement and impaired systolic function of one of both ventricles. Restrictive cardiomyopathy is the least common form in the western hemisphere. The hallmark feature is abnormal diastolic function. The ventricular walls are excessively rigid and impede ventricular filling. Systolic function is often not impaired. Both dilated and restrictive cardiomyopathy can be idiopathic in nature.
Reference:
Kasper, D Braunwald,E et al Harrison's Principles of Internal Medicine 16th ed., McGraw- Hill, New York, 2005 pp1409-1412.
Zipes: Braunwald's Heart Disease: A Textbook of Cardiovascular Medicine, 7th ed, 2005, Elsevier.</p>

How well did you know this?
1
Not at all
2
3
4
5
Perfectly
5
Q
<p>Caution should be given to administering local anesthetics containing vasoconstrictor to hypertensive patients treated with which medication?
A. ACE inhibitors
B. Non-selective beta blockers
C. Calcium channel blockers
D. Angiotensin receptor blockers</p>
A

<p>Answer: B
Rationale:
Treatment of hypertension with a non-selective beta blocker will affect both beta-1 and beta-2 receptors. Epinephrine or levonordefrin will normally cause sympathetic stimulation of both alpha and beta adrenergic receptors. Alpha-1 mediated vasoconstriction is unopposed with beta blockade and can result in severe hypertension and possible reflex bradycardia.
ACE inhibitors affect the renin-angiotensin pathway leading to decreased angiotensin II production. Angiotension II is a potent vasoconstrictor and therefore this reduces peripheral vasoconstriction and afterload and will decrease blood pressure. Ca channel blockers will lead to a decrease in the vasoconstriction of peripheral vasculature leading to a decrease in BP.
Reference:
Ganzberg, Local Anesthetics and Vasoconstrictors. Oral and Maxillofacial Surgery Clinics of North America, Volume 13, #1, p. 71, 2001.</p>

How well did you know this?
1
Not at all
2
3
4
5
Perfectly
6
Q
<p>The agent with the slowest onset of action when treating an acute.hypertensive crisis is:
A. esmolol (Brevibloc).
B. labetolol (Normodyne).
C. sublingual nitroglycerin (Nitrostat).
D. hydralazine (Apresoline).</p>
A

<p>Answer: D
Rationale:
Esmolol is a selective beta blocking agent against B1 receptors. The onset of action for IV esmolol is 2-10 minutes. Labetalol has an onset of 2-5 minutes. It is a B1 and B2 blocker, and also blocks alpha receptors. Onset for sublingual nitroglycerin is 2-5 minutes. Nitroglycerin causes vasodilation and arterial dilation. Hydralazine has an onset of 5-20 minutes. Hydralazine is a direct peripheral dilator, with prolonged duration of action.
Reference:
Dym, The Hypertensive Patient. Oral and Maxillofacial Surgery Clinics of North America, Volume 10, #3, p. 358, 1998.
Stoelting, Dierdorf, Anesthesia and Co-Existing Disease, 4th edition, p. 98, 2002.</p>

How well did you know this?
1
Not at all
2
3
4
5
Perfectly
7
Q
<p>A 56 year-old white male has been diagnosed with secondary hypertension associated with hyperaldosteronism. The laboratory finding consistent with this diagnosis is:
A. hyperkalemia.
B. hypokalemia.
C. hypercalcemia.
D. hypocalcemia.</p>
A

<p>Answer: B
Rationale:
An adrenal adenoma or hyperplastic adrenal gland increases secretion of aldosterone from the zona glomerulosa (outermost layer) of the gland. Primary aldosteronism produces secondary hypertension associated with hypokalemia and suppressed renin activity. Aldosterone causes potassium excretion and sodium resorption from the distal tubule and collecting duct, which leads to the hypokalemia and hypernatremia with increased extracellular fluid volume. Magnesium and chloride levels can also be reflexly altered. Changes in calcium levels are not typically observed.
Reference:
Stoelting, Dierdorf, Anesthesia and Co-Existing Disease, 4th edition, p. 94-95, 429-430, 2002.
Andreoli, et al, Cecil Essentials of Medicine, 5th edition, p.245-246, 572, 2001.</p>

How well did you know this?
1
Not at all
2
3
4
5
Perfectly
8
Q

<p>A 70 year-old male with a history of mitral stenosis presents complaining of longstanding fatigue, exertional dyspnea, and occasional chest pain. Clinically, he shows jugulovenous distension, peripheral edema, and hepatosplenomegaly. Additional clinical findings most consistent with this presentation are:
A. decreased pulmonary artery pressure by catheterization.
B. left ventricular hypertrophy by chest radiograph.
C. ECG findings of right axis deviation, peaked P waves in II, III, and AVF.
D. absent pulmonic component of the second heart sound.</p>

A

<p>Answer: C
Rationale:
Signs and symptoms of cor pulmonale/pulmonary hypertension are described. The ECG findings in answer C are seen with right ventricular and atrial hypertrophy that results. Pulmonary arterial pressure would be elevated in this scenario. LVH may be seen as late sequelae, but not typically. The pulmonic component of the second heart sound is accentuated rather than absent.
Reference:
Stoelting, Dierdorf, Anesthesia and Co-Existing Disease, 4th edition, p. 128, 2002.
Andreoli, et al, Cecil Essentials of Medicine, 5th edition, p.155-156, 2001.</p>

How well did you know this?
1
Not at all
2
3
4
5
Perfectly
9
Q
<p>Which of the following has a progressive PR interval increase?
A. First degree heart block
B. Mobitz I second degree heart block
C. Mobitz II second degree heart block
D. Third degree heart block</p>
A

<p>Answer: B
Rationale:
Mobitz type I second degree AV block is also associated with less than compensatory pause, and a normal QRS duration. It may be seen with drug toxicity such as digitalis and beta blockers.
First-degree heart block, or first-degree atrioventricular (AV) block, is defined as prolongation of the PR interval on the ECG to more than 200 msec.
With first-degree AV block, every atrial impulse is transmitted to the ventricles, resulting in a regular ventricular rate. This type of AV block can arise from delays in the conduction system in the AV node itself, the His-Purkinje system, or a combination of both
Second-degree heart block, or second-degree atrioventricular (AV) block, refers to a disorder of the cardiac conduction system in which some atrial impulses are not conducted to the ventricles. Electrocardiographically, some P waves are not followed by a QRS complex. Second-degree AV block is composed of 2 types: Mobitz I or Wenckebach block, and Mobitz II.
The Mobitz I second-degree AV block is characterized by a progressive prolongation of the PR interval, which results in a progressive shortening of the R-R interval. Ultimately, the atrial impulse fails to conduct, a QRS complex is not generated, and there is no ventricular contraction. The PR interval is the shortest in the first beat in the cycle, while the R-R interval is the longest in the first beat in the cycle.
The Mobitz II second-degree AV block is characterized by an unexpected nonconducted atrial impulse. Thus, the PR and R-R intervals between conducted beats are constant Complete heart block, also referred to as third-degree heart block, or third-degree atrioventricular (AV) block, is a disorder of the cardiac conduction system, where there is no conduction through the AV node. Therefore, complete disassociation of the atrial and ventricular activity exists. The ventricular escape mechanism can occur anywhere from the AV node to the bundle-branch Purkinje system.</p>

How well did you know this?
1
Not at all
2
3
4
5
Perfectly
10
Q

<p>A 64 year-old female is now two days postoperative from a iliac crest graft harvest for a mandibular defect reconstruction. She continues to have dyspnea at rest since emergence from anesthesia. Her B-type natriuretic peptide (BNP) assay is elevated. This may indicate that the patient is suffering from:
A. pulmonary embolism.
B. chronic obstructive pulmonary disease.
C. metabolic acidosis.
D. congestive heart failure.</p>

A

<p>Answer: D
Rationale:
BNP is a neurohormone that is released by the ventricular myocardium in response to ventricular volume and pressure overload. In patients who present with dyspnea of unknown etiology, a plasma BNP > 100 picograms/milliliter (pg/mL) can be used as evidence of heart failure as a cause of the dyspnea (diagnostic accuracy = 84%). This is useful in differentiating dyspnea due to heart failure from noncardiac causes.
Reference:
Marino, L., The ICU Book, Lippincott Williams & Wilkins Philadelphia, 2006, p. 262
Angeja, B.G., Grossman, W., Evaluation and Management of Diastolic Heart Failure, Circulation 2003; 107; 659-663 http://circ.ahajournals.org/cgi/content/full/107/5/659
ACC/AHA 2005 Guideline Update for the Diagnosis and Management of Chronic Heart Failure in the AdultóSummary Article: A Report of the American College of Cardiology/American Heart Association Task Force on Practice Guidelines, Journal of the American College of Cardiology, Volume 46, Issue 6, 20 September 2005, Pages 1122- 1123</p>

How well did you know this?
1
Not at all
2
3
4
5
Perfectly
11
Q
<p>A 70 year-old male is undergoing multiple tooth extractions. Shortly after administering local anesthesia with epinephrine he complains of crushing substernal chest pain that radiates to his left arm. What is the most appropriate initial therapeutic intervention?
A. Sublingual nitroglycerin
B. Supplemental oxygen
C. Crushed aspirin 325 mg PO
D. Intramuscular morphine sulfate</p>
A

<p>Answer: B
Rationale:
The patient being treated is likely suffering from an ischemic myocardial injury as described with the symptom of substernal chest pain. Despite the common acronym MONA, the initial treatment of ischemic heart disease is oxygen, nitroglycerin, aspirin, and morphine, in that order. Oxygen administration may limit ischemic myocardial injury, although its effects on morbidity and mortality of acute infarction are unknown. A short period of initial routine oxygen supplementation is reasonable during initial stabilization of the patient, given its safety and the potential for underrecognition of hypoxemia. Farther down the chain of early treatment of chest pain, a 12 lead ECG would be obtained early in the assessment of his chest pain. Later, cardiac marker enzymes levels (Troponin T, Troponin I, CK-MB) would be used in the diagnosis of myocardial injury.
Reference:
American Heart Association: Handbook of Emergency Cardiovascular Care for Healthcare Providers, American Heart Association, Dallas Texas, 2006, pages 22 - 28
ACC/AHA 2007 Guidelines for the Management of Patients With Unstable Angina/Nonñ ST-Elevation Myocardial Infarction: A Report of the American College of Cardiology/American Heart Association Task Force on Practice Guidelines (Writing Committee to Revise the 2002 Guidelines for the Management of Patients With Unstable Angina/NonñST-Elevation Myocardial Infarction), Journal of the American College of Cardiology, Volume 50, Issue 7, 14 August 2007, page 39</p>

How well did you know this?
1
Not at all
2
3
4
5
Perfectly
12
Q

<p>What are the beneficial effects of using ACE inhibitors and diuretics in combination for the treatment of congestive heart failure?
A. Positive chronotropic and inotropic effects
B. Negative chronotropic and inotropic effects
C. Increase preload and afterload
D. Decrease preload and afterload
</p>

A

<p>Answer: D
Rationale:
ACE inhibitors and diuretics are recommended for routine use in treating CHF.
ACE inhibitors block the renin-angiotensin-aldosterone system producing vasodilation by limiting angiotensin II-induced vasoconstriction. The vasodilation is predominantly arterial which decreases afterload. This decreases myocardial work and decreases myocardial energy requirements.
Diuretics decrease extracellular fluid thereby decreasing ventricular filling pressures or preload. This treats the symptoms of CHF.
Reference:
ACC/AHA 2005 Guideline Update for the Diagnosis and Management of Chronic Heart Failure in the AdultóSummary Article: A Report of the American College of Cardiology/American Heart Association Task Force on Practice Guidelines, Journal of the American College of Cardiology, Volume 46, Issue 6, 20 September 2005, Pages 1128- 1129
Cooper, D., The Washington Manual of Medical Therapeutics, 32nd Edition, Chapter 6, pages 171 and 174, 2007</p>

How well did you know this?
1
Not at all
2
3
4
5
Perfectly
13
Q

<p>Listen to the attached audio clip and identify the cause of the heart murmur. A. Aortic valve regurgitation B. Mitral valve regurgitation C. Tricuspid valve regurgitation D. Ventricular septal defect</p>

A

<p>Answer: A Rationale: Tricuspid valve regurgitation, mitral valve regurgitation, and ventricular septal defects all cause murmurs that are audible during the systolic phase of heart function. Aortic valve regurgitation occurs during the diastolic phase of heart function (represented by the longer time period between heart sounds) and most frequently occurs due to aortic root dilation or rheumatic heart disease. A portion of the LV stroke volume is expelled during systole and regurgitates in the LV during diastole leading to AR regurgitation. Aortic regurgitation occurs when the aortic valve fails to close completely and blood flows back into the left ventricle after ejection into the aorta is complete (after S2). Normally, there is a brief period of time after the aortic valve closes when the ventricle relaxes isovolumetrically (the mitral valve is also closed during this phase). But when the aortic valve is leaky, the ventricle begins to fill from the aorta after the incomplete closure of the aortic valve. This leads to an increase in ventricular volume prior to the opening of the mitral valve and normal ventricular filling. Because blood is leaving the aorta in two directions (back into the heart as well as down the arterial network), the aortic diastolic pressure falls more rapidly thereby leading to a decrease in arterial diastolic pressure. Because the ventricle fills from both the aorta and the left atrium, there is a large increase in left ventricular volume and pressure (increased preload), which is best depicted by pressure-volume loops for this condition. The increased preload causes the left ventricle to contract more forcefully (Frank-Starling mechanism), thereby increasing ventricular (and aortic) systolic pressure and increasing stroke volume to help compensate for the regurgitation. The increase in ventricular end-diastolic pressure, however, also leads to an increase in left atrial pressure, which can result in pulmonary congestion and edema. Regurgitation, coupled with enhanced left ventricular stroke volume, results in a characteristic widening of the aortic pulse pressure. The backward flow of blood into the ventricular chamber during diastole results in a diastolic murmur between S2 and S1.</p>

How well did you know this?
1
Not at all
2
3
4
5
Perfectly
14
Q
<p>Which of the following is the risk factor for sudden cardiac death in a patient with aortic stenosis?
A 1%
B. 5%
C. 10%
D. 15%</p>
A

<p>Answer: B
Rationale:
Aortic stenosis (AS) is the obstruction of blood flow across the aortic valve. AS has several etiologies: congenital unicuspid or bicuspid valve, rheumatic fever, and degenerative calcific changes of the valve.
Pathophysiology: When the aortic valve becomes stenotic, resistance to systolic ejection occurs and a systolic pressure gradient develops between the left ventricle and the aorta. Stenotic aortic valves have a decreased aperture that leads to a progressive increase in left ventricular systolic pressure. This leads to pressure overload in the left ventricle, which, over time, causes an increase in ventricular wall thickness (ie, concentric hypertrophy). At this stage, the chamber is not dilated and ventricular function is preserved, although diastolic compliance may be affected.
Eventually, however, the left ventricle dilates. This, coupled with a decrease in compliance, is associated with an increase in left ventricular end-diastolic pressure, which is increased further by a rise in atrial systolic pressure. A sustained pressure overload eventually leads to myocardial decompensation. The contractility of the myocardium diminishes, which leads to a decrease in cardiac output. The elevated left ventricular end-diastolic pressure causes a corresponding increase in pulmonary capillary arterial pressures and a decrease in ejection fraction and cardiac output. Ultimately, congestive heart failure (CHF) develops.
In the US: This is a relatively common congenital cardiac defect. Incidence is 4 in 1000 live births.
Mortality/Morbidity: Sudden cardiac death occurs in 3-5% of patients with AS. Adults with AS have a 9% mortality rate per year. Once symptoms develop the incidence of sudden death increases to 15-20%, with average survival duration of less than 5 years. Patients with exertional angina or syncope survive an average of 3 years. After the development of left ventricular failure, life expectancy is slightly greater than 1 year.
</p>

How well did you know this?
1
Not at all
2
3
4
5
Perfectly
15
Q

<p>The above EKG shows which of the following rhythms?</p>

<p>A. Ventricular escape rhythm B. First degree heart block C. Second degree heart block D. Third degree heart block</p>

A

<p>Answer: D Rationale: The ECG with third degree heart block has the following three characteristics: P waves are present with a regular atrial rate faster than the ventricular rate, QRS complexes are present with a slow ventricular rate, and the P waves bear no relation to the QRS complexes, and the PR intervals are completely variable because the atria and ventricles are electrically disconnected. First-degree AV block, defined as a PR interval exceeding 200 milliseconds in an adult (180 milliseconds in adolescents), is more accurately described as first-degree AV conduction delay. Second degree heart block is divided into Mobitz I and Mobitz II. Mobitz type I, also called Wenckebach, results in progressive lengthening of the P-R interval with eventual drop of a QRS complex. Mobitz II is characterized by the sudden loss of a QRS complex without P-R elongation. Reference: Goldberger: Clinical Electrocardiography: A Simplified Approach, 7th ed., Copyright © 2006 Mosby, An Imprint of Elsevier Rakel: Conn's Current Therapy 2006, 58th ed., Copyright © 2006 Saunders, An Imprint of Elsevier</p>

How well did you know this?
1
Not at all
2
3
4
5
Perfectly
16
Q

<p>Below is a gram stain of drainage from a neck wound. Which of the following is the most appropriate initial oral antibiotic therapy?</p>

<p>(Picture-Gram positive cocci)</p>

<p>Metronidazole</p>

<p>Trimethoprim-sulfamethoxazole</p>

<p>Penicillin</p>

<p>Vancomycin</p>

A

<p>Answer: B</p>

<p>Rationale:<br></br>
The gram stain of gram-positive cocci in clusters is consistent with staphylococcus, which is aerobic. Metronidazole has antimicrobial activity against anaerobic bacteria. Penicillin has some antimicrobial against staphylococcus that does not produce penicillinase. The gram stain does not provide any insight as to whether the bacteria produce penicillinase, so this would be a poor choice. Vancomycin does have excellent antibacterial properties against staphylococcus, but it is not absorbed when taken orally. Trimethoprim- sulfamethoxazole, although typically prescribed for its activity against gram negative bacteria, also has excellent antibacterial properties against staphylococcus, is well absorbed orally, and is the most appropriate initial antibiotic of those listed.</p>

<p>Reference:<br></br>
McCarter, YS In Topazian, RG, Goldberg, MH, Hupp, JR, editors: Oral and Maxillofacial Infections, ed 4, Philadelphia, 2002, W B Saunders Co., pp 47-50.</p>

<p>Hupp, JR In Topazian, RG, Goldberg, MH, Hupp, JR, editors: Oral and Maxillofacial Infections, ed 4, Philadelphia, 2002, W B Saunders Co., pp 112-125.</p>

How well did you know this?
1
Not at all
2
3
4
5
Perfectly
17
Q

<p>A patient with a severe cervicofacial infection is noted to have tea-colored urine. The presence of which of the following on urinalysis would suggest necrotizing fasciitis?</p>

<p>Bilirubin</p>

<p>Hemoglobin</p>

<p>Myoglobin</p>

<p>Haptoglobin</p>

A

<p>Answer: C</p>

<p>Rationale:<br></br>
Necrotizing fasciitis can be associated with rhabdomyolysis and secretion of myoglobin in the urine. Although severe infections can be associated with hemolysis and subsequent hemoglobinuria, and increased urine bilirubin, these parameters are not suggestive of underlying muscle inflammation and necrosis. Haptoglobin binds hemoglobin and is too large a molecule to be filtered n the urine.</p>

<p>Reference:<br></br>
Marx: Rosen's Emergency Medicine: Concepts and Clinical Practice, 5th ed., Copyright © 2002 Mosby, Inc. pp.1763-1766</p>

<p>Hoffman: Hematology: Basic Principles and Practice, 3rd ed., Copyright © 2000 Churchill Livingstone, Inc., p. 408.</p>

How well did you know this?
1
Not at all
2
3
4
5
Perfectly
18
Q

<p>Antibiotic prophylaxis prior to dentoalveolar surgery is recommended for which disorder?</p>

<p>A. Mitral valve prolapse</p>

<p>B. Mitral valve prolapse with regurgitation</p>

<p>C. Artificial heart valves</p>

<p>D. Calcified aortic stenosis</p>

A

<p>Answer: C</p>

<p>Rationale:<br></br>
According to the newest AHA guidelines (11) from (4-19-2007):<br></br>
Antiobiotic prophylaxis is required prior to dental surgery only for the following conditions:</p>

<p>1. 2. 3.</p>

<p>Artificial heart valves<br></br>
A history of having had bacterial endocarditis<br></br>
Certain specific, serious congenital (present from birth) heart conditions, including:</p>

<p>a) Unrepaired or incompletely repaired cyanotic congenital heart disease, including those with palliative shunts and conduits</p>

<p>b) A completely repaired congenital heart defect with prosthetic<br></br>
Material or device, whether placed by surgery or by catheter intervention, during the first six months after the procedure</p>

<p>c) Any repaired congenital heart defect with residual defect at the site or adjacent to the site of a prosthetic patch or a prosthetic device</p>

<p>d) A cardiac transplant which develops a problem in a heart valve.</p>

<p></p>

<p>Reference:<br></br>
Wilson W, Taubert KA, Gewitz M, Lockhart PB, Baddour LM, Levison M, Bolger A, Cabell CH, Takahashi M, Baltimore RS, Newburger JW, Strom BL,<br></br>
Tani LY, Gerber M, Bonow RO, Pallasch T, Shulman ST, Rowley AH, Burns<br></br>
JC, Ferrieri P, Gardner T, Goff D, and Durack DT. Prevention of Infective Endocarditis. Guidelines from the American Heart Association.<br></br>
A Guideline from the American Heart Association Rheumatic Fever,<br></br>
Endocarditis, and Kawasaki Disease Committee, Council on Cardiovascular<br></br>
Disease in the Young, and the Council on Clinical Cardiology, Council on Cardiovascular Surgery and Anesthesia, and the Quality of Care and Outcomes Research Interdisciplinary Working Group. Circulation 2007</p>

How well did you know this?
1
Not at all
2
3
4
5
Perfectly
19
Q

<p>Which factor decreases a patientís risk for post-operative mandibular fracture after third molar removal?</p>

<p>A. Female gender</p>

<p>B. Age greater than 25 years-old</p>

<p>C. Full dentition</p>

<p>D. Distoangular position of the third molar</p>

A

<p>Answer: A</p>

<p>Rationale:<br></br>
The main factors that increase the risk for post-operative mandible fracture after third molar removal are: 1) age greater than 25 years-old</p>

<p>2) male gender 3) full dentition</p>

<p>Reference:<br></br>
Libersa P, Roze D, Cachart T, Libersa JC. Immediate and late mandibular fractures after third molar removal. J Oral Maxillofac Surg. 2002; 60:163-165; discussion 165-166.</p>

<p>Krimmel M, Reinert S. Mandibular fracture after third molar removal. J Oral Maxillofac Surg. 2000; 58:1110-1112.</p>

How well did you know this?
1
Not at all
2
3
4
5
Perfectly
20
Q

<p>Which of the following best characterizes the basement membrane surface of Alloderm (acellular dermal matrix), when used as an interpositional graft for root coverage?</p>

<p>A. Retains reddish coloration after contact with the patientís blood</p>

<p>B. Facilitates epithelial cell migration</p>

<p>C. Should be placed away from the exposed surface intended for coverage</p>

<p>D. Promotes revascularization</p>

A

<p>Answer: B</p>

<p>Rationale:<br></br>
The basement membrane surface of the Alloderm graft facilitates epithelial cell migration and attachment. The connective tissue side contains vascular channels that allow for cellular infiltration and revascularization. The basement membrane surface should be placed in contact with the exposed root surface when attempting to achieve root coverage, as this is an epithelial surface. The connective tissue surface retains a reddish coloration after contact with the patient's blood; the basement membrane side remains white.</p>

<p>Reference:<br></br>
Miloro M et al, Peterson's Principles of Oral and Maxillofacial Surgery. Pages 220-221, Second Edition, BC Decker, 2004.</p>

<p>Babbush CA et al, Dental Implants. The Art and Science. Implant Periabutment Tissue. Pages 127-129, WB Saunders, 2001.</p>

How well did you know this?
1
Not at all
2
3
4
5
Perfectly
21
Q

<p>A semilunar flap technique for coverage of a root with marginal tissue recession is indicated in which of the following situations?</p>

<p>A. Absence of interdental papilla</p>

<p>B. Thin scalloped periodontium</p>

<p>C. Lack of severe facial ridge curvature</p>

<p>D. Inadequate zone of keratinized tissue</p>

A

<p>Answer: C</p>

<p>Rationale:<br></br>
A semilunar flap technique will not recreate the dental papilla and it exhibits poor success in patients with thin scalloped periodontium. If there is not an adequate zone of keratinized tissue to start, the procedure cannot be accomplished. A severe facial curvature of the bone will also prevent success of the technique.</p>

<p>Reference:<br></br>
Nasr H, Atlas of the Oral and Maxillofacial Surgery Clinics of North America, Vol 7 Number 2, Sept 1999, pg 29-37</p>

<p>Tarnow, D, Solving Restorative Esthetic Dilemmas with the Semilunar Coronally Positioned Flap Journal of Esthetic and Restorative Dentistry 6 (2) 1994, 61ñ64.</p>

How well did you know this?
1
Not at all
2
3
4
5
Perfectly
22
Q

<p>Which of the following is a limitation of the palatal connective tissue graft technique?</p>

<p>A. High incidence of poor healing</p>

<p>B. Dependence on smooth palate donor site</p>

<p>C. Graft availability is dependent on donor site thickness</p>

<p>D. High incidence of neurovascular injury</p>

A

<p>Answer: C</p>

<p>Rationale:<br></br>
Connective tissue grafting extremely useful in that is does not depend upon a smooth palate and heals very well. The incidence of neurovascular injury is also very low if harvested in the classic manner (anterior to the maxillary first molar.) Depending upon the thickness of a particular patient's tissue, the amount of graft available may be minimal and therefore some patients may require secondary grafting several months later.</p>

<p>Reference:<br></br>
Sclar A, Alpha Omegan, Volume 93, number3, Aug/Sept 2000, pg 38-46.</p>

<p>Fonseca, et al. Oral and Maxillofacial Surgery: Reconstruction and Implant Surgery. (Vol 7). WB Saunders Company. Philadelphia. 2000, pp 335-8.</p>

How well did you know this?
1
Not at all
2
3
4
5
Perfectly
23
Q

A trauma patient in the surgical intensive care unit has the following blood gas result.
pH 7.32 (normal range 7.36 ñ 7.44)
PaCO2 46 mm Hg (normal range 36-44 mm Hg) HCO3 23 mEq/L (normal range 22-26 mEq/L)
This finding is indicative of which condition?
A. Respiratory acidosis
B. Respiratory alkalosis
C. Metabolic acidosis
D. Metabolic alkalosis

A

Answer: A
Rationale:
Respiratory acidosis occurs when the pH is below 7.36 and PaCO2 is above 44 mm/Hg. Respiratory alkalosis occurs when the pH is above 7.44 and PaCO2 is below 36 mm/Hg. Metabolic acidosis has a pH below 7.36 and HCO3 is below 22 mm/Hg. Metabolic alkalosis has a pH above 7.44 and HCO3 is higher than 26 mm/Hg.
Respiratory acidosis occurs with impairment in the rate of alveolar ventilation. Acute respiratory acidosis occurs with a sudden depression of the medullary respiratory center, paralysis of the respiratory muscles, and with airway obstruction. Therapy is aimed at treatment of the underlying disorder and ventilatory support. The question above refers to uncompensated respiratory acidosis. The body does use metabolic and respiratory compensatory mechanisms to maintain a constant PaCO2/ HCO3 ratio. For example in a primary respiratory acidosis or alkalosis, the kidneys provide the compensation by adjusting the HCO3 reabsorption. In a primary metabolic disorder, the ventilatory system is mediated by H+ sensitive chemoreceptors in the carotid body which signal an increase or decrease in ventilation to alter arterial PaCO2 levels. The amount of compensation can be calculated using the pH, PaCO2, and bicarbonate level.
Respiratory alkalosis occurs with hyperventilation. It can occur in pregnancy, fever and septic states, with pneumonia, pulmonary embolism, and congestive heart failure. Acute hyperventilation is characterized by light-headedness, paresthesia, circumoral numbness, and tingling of the extremities. Tetany occurs in severe cases.
Metabolic acidosis can be due to extrarenal loss of bicarbonate, as in diarrheal diseases, but can also be caused by high renal excretion of bicarbonate.
Metabolic alkalosis is caused by things such as excessive vomiting and diuretic use leading to volume depletion. It occurs due to a failure to the kidney to excrete excess bicarbonate. Treatment is directed at correction of the metabolic disorder.
American Board of Oral and Maxillofacial Surgery
18
2008 Oral and Maxillofacial Surgery Self Assessment Tool (OMSSAT)
Reference:
The ICU Book, 2nd ed., Marino, Williams and Wilkins, 1997 p. 584-586
Cecil Essentials of Medicine, 7th ed. Andreoli and Carpenter, Saunders, 2007, 298-303.

How well did you know this?
1
Not at all
2
3
4
5
Perfectly
24
Q
Which of the following is the most important risk factor for developing nosocomial pneumonia?
A. Malnutrition
B. Mechanical ventilation
C. Nursing home residence
D. Tobacco abuse
A

Answer: B
Rationale:
While malnutrition, tobacco use, and residency in a nursing home facility are risk factors, endotracheal intubation and mechanical ventilation is by far the most important contributor leading to nosocomial pneumonia. Endotracheal intubation provides a pathway for bacterial contamination in the lungs.
This is increased in the patient with chest trauma or lung injury. The aspiration of oral secretions into the upper airways is the inciting event in most cases of pneumonia. Averages of 1 billion bacteria are found in each milliliter of saliva.
Reference:
Kokko, J., Stein, S., The Emory University Comprehensive Board Review in Internal Medicine. McGraw-Hill. New York, 2000, p. 286.
Mandell, Bennett, & Dolin: Principles and Practice of Infectious Diseases, 6th ed., Chapter 314 ñ Infections in the Elderly, Pneumonia, Copyright © 2005 Churchill Livingstone, An Imprint of Elsevier
Marino, P., The ICU Book. Lippincott Williams Wilkins. Baltimore, 2004, p. 516-517.

How well did you know this?
1
Not at all
2
3
4
5
Perfectly
25
Q
Chronic complications of which disease process includes cirrhosis of the liver, pancreatic dysfunction, sinusitis, and bronchial hyperreactivity?
A. Chronic bronchitis
B. Cystic fibrosis
C. Sarcoidosis
D. Tuberculosis
A

Answer: B
Rationale:
Cystic fibrosis is an autosomal recessive disorder. In the Untied States, it affects 1 in 2500 Caucasians, 1 in 17000 African Americans, and 1 in 90000 Asians. Over 230 mutations have been identified with various degrees of disease severity, with the most common defect on the long arm of chromosome 7. The genetic defect causes defective chloride transport and increased sodium reabsorption in airway and ductal epithelia, creating abnormally thick and viscous secretions in the respiratory, hepatobiliary, gastrointestinal, and reproductive tracks. The thick secretions cause luminal obstruction and destruction of exocrine ducts. The median survival age is 40 years.
The disease often manifests in childhood. Respiratory complications in addition to chronic sinusitis and bronchial hyperreactivity include mucous plugging within the lungs. Mucous plugs can also occur in the salivary glands, although not common.
A diagnostic criterion is a sweat chloride concentration > 60 mEq/L. Genetic testing can also be completed to confirm the diagnosis. Clinical findings are also important in diagnosis. Presence of mucoid Pseudomonas aeruginosa in sputum is characteristic of CF. Chest radiograph abnormalities include bronchiectasis with upper lobe and right-sided predominance. Digital clubbing and hypertrophic osteoarthropathy are present. Spontaneous pneumothorax and pulmonary hypertension are negative prognostic findings.
Standard therapy for CF includes chest physical therapy, antibiotics as needed, adequate nutrition, and exercise. Bronchodilators may be used for patients with bronchospasm or airflow limitation. Lung transplants for patients with advanced CF are being performed.
Reference:
Kokko, J., Stein, S., The Emory University Comprehensive Board Review in Internal Medicine. McGraw-Hill. New York, 2000, p. 305-306.
Andreoli T: Cecil Essentials of Medicine, W. B. Saunders Company, Philadelphia, 2001, p. 220-221.

How well did you know this?
1
Not at all
2
3
4
5
Perfectly
26
Q
Which of the following is commonly associated with asthma?
A. Decreased lung compliance
B. Obtunded cholinergic sensitivity
C. Gastroesophageal reflux disease
D. Hypocarbia
A

Answer: C
Rationale:
Upwards of 80% of asthmatics exhibit gastroesophageal reflux disease (GERD). Although respiratory disease can induce GER by increasing abdominothoracic pressure gradients, there is substantial evidence suggesting that the more common pathway is that for GER to trigger respiratory symptoms, particularly of asthma. This may occur from microaspiration with resultant direct inflammation and bronchoconstriction, from vagally mediated effects from stimulation of upper airway receptors, or from esophageal afferent irritation causing a neurogenic reflex.
Decreased lung compliance is associated with restrictive lung disease not obstructive, and asthma is an obstructive pulmonary disease.
Asthmatics commonly have increased cholinergic sensitivity.
Hypercarbia is sequelae of an asthmatic attack.
Reference:
Behrman: Nelson Textbook of Pediatrics, 17th ed., Copyright © 2004 Saunders, An Imprint of Elsevier, Chapter 387, Gastroesophageal reflux and respiratory disorders.
Mason: Murray and Nadel’s Textbook of Respiratory Medicine, 4th ed. Chapter 78 ñ Pulmonary Complications of Abdominal Disease.

How well did you know this?
1
Not at all
2
3
4
5
Perfectly
27
Q
Which of the following best characterize restrictive lung disease?
A. Decrease in total lung capacity
B. Decrease in FEV1/FVC ratio
C. Increase in vital capacity
D. Increase in airway resistance
A

Answer: A
Rationale:
Restrictive disorders are best evaluated by measuring lung volumes. The severity of the restrictive defect is based on the TLC (total lung capacity). In restrictive lung disorders, the FEV1 may be decreased, but the FEV1/FVC ratio will be preserved. The TLC, FRC (functional residual capacity) and RV (residual volume) will all be decreased. Airway resistance is not affected in a restrictive lung disease.
Restrictive disorders can occur in three circumstances: lung disorders, disorders of the chest wall, and neuromuscular disease. Lung disorders with interstitial infiltration typically show restriction caused by increased elastic recoil such as observed in idiopathic pulmonary fibrosis and sarcoidosis. Lung edema as a result of congestive heart failure can also cause a restrictive pattern. Chest wall abnormalities may exhibit themselves with a restrictive pattern by restricting lung expansion as observed in kyphoscoliosis, obesity , and ankylosing spondylitis, Included in the chest wall abnormalities are the pleural diseases (pleural effusion, pneumothorax,), space occupying lesions (tumors), and conditions causing increased abdominal girth such as pregnancy, ascites, and large intra-abdominal tumors. Some neuromuscular disorders cause restriction by preventing normal excursion of the lung during breathing as observed in patients with myasthenia gravis, amyotrophic lateral sclerosis, diaphragmatic paralysis, and the Guillain-Barre syndrome. Patients may also restrict lung excursion during inspiration as a result of pain or somnolence (drug overdose). Lung resection during lobectomy will also cause a restrictive defect.
Decrease in the FEV1/FVC ratio is the hallmark of obstructive lung disease. Airway resistance is increased in obstructive lung disease (asthma).
Reference:
Andreoli T: Cecil Essentials of Medicine, W. B. Saunders Company, Philadelphia, 2001, p. 208-210.
Clinical Anesthesiology 4th ed., Lange Medical Books/McGraw Hill, New York, 2006 pp572-580

How well did you know this?
1
Not at all
2
3
4
5
Perfectly
28
Q
What is the most likely diagnosis for a patient with the following findings after pulmonary function testing?
FVC - normal
FEV1 - decreased FEV1/FVC - decreased
A. Respiratory muscle weakness
B. Obstructive lung disease
C. Restrictive lung disease
D. Spontaneous pneumothorax
A

Answer: B
Rationale:
Obstructive lung disease includes asthma, chronic bronchitis, emphysema, bronchiectasis, cystic fibrosis, and upper airway obstruction.
Diseases such as asthma and bronchitis, which obstruct the airway, reduce expiratory flow rates and therefore reduce FEV1 and FEV1/FVC. An obstructive defect is essentially characterized by a disproportionate decrease in the airflow rate relative to the actual volume exhaled (i.e., FVC) and indicates airway narrowing and flow limitation during expiration. Values for FEV1/FVC that are lower than 70% reflect mild obstruction, those lower than 60% suggest moderate obstruction, and those lower than 50% indicate severe obstruction.
In restrictive lung disease and respiratory muscle weakness, the forced vital capacity and FEV1 are decreased, while the FEV1/FVC ratio is normal. Pneumothorax can be considered a restrictive lung disease.
Reference:
Andreoli T: Cecil Essentials of Medicine, W. B. Saunders Company, Philadelphia, 2001, p. 208-210.
Kokko, Jl, Stein,S., The Emory University Comprehensive Board Review in Internal Medicine. Pg 271-283, 301-311 McGraw-Hill. New York, 2000.

How well did you know this?
1
Not at all
2
3
4
5
Perfectly
29
Q

Which is considered the most definitive in the diagnosis of pulmonary embolism?
A. Ultrasound of femoral-popliteal system
B. Nuclear ventilation-perfusion scan
C. Chest radiograph
D. Contrast-enhanced CT scan

A

Answer: D
Rationale:
Pulmonary embolism (PE) is usually a complication of venous thrombosis in the deep veins of the lower extremities that originate in the femoral-iliac-pelvic veins. Causes of thrombosis include: vessel injury, hypercoagulability, and stasis. The best treatment of PE is prevention, especially in high risk patients (malignancy, obesity, age greater than 40, PE history, extensive surgery, and immobilization). In hemodynamically unstable patients it is important to definitively diagnose and treat. The test that was considered the ìgold standardî for many years was pulmonary angiogram. Helical or multi-detector CT scans are now becoming the preferred test for definitive diagnosis.
In hemodynamically stable patients the following is a suggested treatment protocol.
1) Maintain high index of suspension
2) Signs and symptoms: - dyspnea, tachypnea, tachycardia, chest pain, PO2 less than 80 (none is completely diagnostic)
3) Chest XR: may show localized volume loss, atelectasis, peripheral infiltrates, and/or pleural effusions
4) Ventilation-perfusion scan (V/Q): This scan uses lung ventilation with radiolabeled tracer gas with lung perfusion by radiolabeled microocclusive particles. A normal scan will rule out PE. Results of a low or intermediate scan gives likelihood between 15-55% and usually indicates pulmonary angiogram for conformation. The results of V/Q are dependent on quality of the scan, interpretation of the scan, and clinical probability of PE. 5) Impedance plethysmography and Doppler ultrasonograph are noninvasive techniques for diagnosis of deep vein thrombosis in the lower extremity that may support diagnosis of PE. These tests have a low sensitivity. Venous duplex scanning (ultrasonic venous imaging with Doppler blood-flow imaging) has greater accuracy than the above procedures.
6) Contrast-enhanced CT is increasingly used as the initial radiologic study in the diagnosis of PE, especially in patients with abnormal chest radiographs in whom scintigraphic results are more likely to be nondiagnostic.
CT show emboli directly, as does pulmonary angiography, and it is also noninvasive, cheaper, and widely available. CT is the only test that can provide significant additional
information related to alternate diagnoses; this is a clear advantage of CT compared with either pulmonary angiography or scintigraphy.
Because DVT and PE are part of the same disease process, CT venography can easily be performed after CT pulmonary angiography.
When PE is identified, it is characterized as acute or chronic. An embolus is acute if it is situated centrally within the vascular lumen or if it occludes a vessel (vessel cutoff sign). Acute PE commonly causes distention of the involved vessel. An embolus is chronic if (1) it is eccentric and contiguous with the vessel wall, (2) it reduces the arterial diameter by more than 50%, (3) evidence of recanalization within the thrombus is present, and (4) an arterial web is present.
PE is further characterized as central or peripheral, depending on the location or the arterial branch involved. Central vascular zones include the main pulmonary artery, the left and right main pulmonary arteries, the anterior trunk, the right and left interlobar arteries, the left upper lobe trunk, the right middle lobe artery, and the right and left lower lobe arteries. Peripheral vascular zones include the segmental and subsegmental arteries of the right upper lobe, the right middle lobe, the right lower lobe, the left upper lobe, the lingula, and the left lower lobe.
7) Pulmonary angiogram: The last test in this protocol because of its invasive nature and inherent risk. The role of conventional angiograghy is limited to patients in which other results are non-diagnostic and/or clinical suspicions are high.
D-dimer: Blood test that is used as an aid to diagnose thrombosis. It is best used to rule out thromboembolic disease when probability is low. A negative result practically rules out thrombosis and a positive test may indicate thrombosis but does not rule out other etiologies (DIC) and requires further testing for DVT/PE.
D-dimer is unique in that they are breakdown products of fibrin mesh after being stabilized/cross-linked by Factor XIII. This is the final step in the generation of a thrombosis and indicate a thrombosis is present.
Reference:
Kokko, J. Stein, S., The Emory University Comprehensive Board Review in Internal Medicine. Pg 327-331 McGraw-Hill New York, 2000.
Marino, P The ICU Book. Pg 106-115, 376-379 Lippincott Williams Wilkins. Baltimore, 2006.
Eng, j. Krishman, JA. Segal, J. Bolger, D. Tamariz, L. Accuracy of CT in the Diagnosis of Pulmonary Embolism. Journal of Roentololgy. 2004; 183:1819-1827.

How well did you know this?
1
Not at all
2
3
4
5
Perfectly
30
Q
Which of the following pharmacologic groups is a risk factor for venous thromboembolism?
A. Hypercholesterolemics
B. Estrogen replacement therapy
C. Non steroidal anti-inflammatories
D. ACE inhibitors
A

Answer: B
Rationale:
Estrogens and birth control pills are can result in hypercoagulability and are a risk factor for venous thromboembolism Pulmonary embolism is usually a complication of venous thrombosis of the femoral-iliac-pelvic-popliteal system of the lower extremity. Factors that may contribute to thrombosis include: venous stasis, hypercoagulability, endothelial injury. Patient risk factors include: heart failure, malignancy, immobilization, trauma, obesity, advanced age, pregnancy, extensive surgery, and oral contraceptives.
There is no data to support hypercholerterolemics, NSAID’s or ACE inhibitors as risk factors for venous thromboembolism.
Prevention is the best treatment which includes: intermittent compression device, anti- embolism stocking, selective heparin use, and early post-operative ambulation.
Reference:
Kokko, J., Stein, S., The Emory University Comprehensive Board Review in Internal Medicine. McGraw-Hill. New York, 2000, p.329-331.
Marino, P. The ICU Book. Lippincott Williams Wilkins. Baltimore, 2006, p. 106-108.

How well did you know this?
1
Not at all
2
3
4
5
Perfectly
31
Q
What is the first course of action for patients strongly suspected or documented to have a deep vein thrombosis (DVT)?
A. Warfarin
B. Thrombectomy
C. Heparin
D. Streptokinase
A

Answer: C
Rationale:
Treatment should include supportive care for hypoxemia and hypotension. Anticoagulation with IV heparin should be started as soon clinical suspicion of PE without waiting for diagnostic studies. The goal is for 1.5-2.5 times the control value of PTT for 5-10 days. Any delay in reaching therapeutic PTT range has been shown to increase the risk of progressive thrombosis and recurrent pulmonary embolism. Then, warfarin therapy is initiated with an INR goal of 2-3 for 3-6 months or indefinitely if patient is at high risk. If contraindications exist for anticoagulation or if patient has reoccurrence of PE in spite of anticoagulation the treatment of choice is vena caval interruption. (Greenfield filter). Systemic thrombolytic therapy and embolectomy may be indicated in very rare cases.
Heparin (unfractionated) is a glycosaminoglycan with varying molecular size. It potentiates the action of antithrombin III thereby inactivating thrombin and prevents the conversion of fibrinogen to fibrin.
Low molecular weight heparin (LMWH) has been used in place of the unfractionated heparin with comparable results for treatment of DVT’s and non-massive PE. LMWH is enzymatically broken down into smaller more uniform molecules with less variation in anti-coagulation response. Because of the predictability of the response and route of administration these patients are, in some cases, treated as outpatients, dramatically reducing cost but not changing clinical outcomes. LMWH is given subcutaneously with a slower onset then IV heparin but dosing is less frequent, there is less heparin-induced thrombocytopenia, less heparin-induced osteoporosis, and slightly less risk of bleeding.
Reference:
Green, GB, Harris, IS., Lin, GA., Moylan, KC., The Washington Manual of Medical Therapeutics, 31st Edition. Lippincott Williams Wilkins. Baltimore, 2004.
Marino, P., The ICU Book. Pg 112-115, 376-379 Lippincott Williams Wilkins. Baltimore, 2004.

How well did you know this?
1
Not at all
2
3
4
5
Perfectly
32
Q
A 60 year-old patient reports a history of smoking and frequent productive cough present for at least three months in each of the last two years. What is the most likely diagnosis?
A. Emphysema
B. Cystic fibrosis
C. Asthma
D. Chronic bronchitis
A

Answer: D
Rationale:
Chronic bronchitis is defined as the presence of a productive cough for at least 3 months in each of the past 2 years in a person with excessive mucous secretion not due to other disease. The most common cause is cigarette smoking. Chronic bronchitis may result in carbon dioxide retention, chronic hypoxemia, erythrocytosis, pulmonary hypertension and right heart failure. The respiratory drive becomes less sensitive to carbon dioxide retention and may potentially be depressed by oxygen administration. (Blue Bloater Syndrome)
Emphysema is characterized by irreversible enlargement of the airways distal to the terminal bronchioles and destruction of the alveolar septa. Emphysema will usually have little sputum production and a cough with exertion.
Asthma is characterized by airway inflammation and hyper-reactivity in response to various stimuli. Asthma produces episodic attacks of dyspnea, cough, and wheezing. Cystic fibrosis is characterized by secretion of highly viscous mucous with fibrosis and chronic lung infections.
Reference:
Clinical Anesthesiology 4th ed., Lange Medical Books/McGraw Hill, New York, 2006 pp572-580
Rita K Cydulka: Emergency Medicine 6th ed., McGraw Hill, New York, 2004 pp 475-480

How well did you know this?
1
Not at all
2
3
4
5
Perfectly
33
Q

Which of the following pulmonary function tests is most useful in evaluating the asthmatic patient?
A. TLC (Total lung capacity)
B. FEV1 (Forced expiratory volume in 1 second)
C. FRC (Functional residual capacity)
D. VC (Vital capacity)

A

Answer: B
Rationale:
The FEV1 measures the amount of air which can be forcibly expelled in 1 second and reflects the degree of large airway obstruction. FEV1 values are normally around 3L for men and 2L for women, but vary with each individual. FEV1/FVC should normally be >70% and an FEV1 or FEV1/FRC ratio less than 50% indicates moderate to severe asthma. TLC and FRC may be elevated in obstructive disease. Obstructive disease (such as asthma) is characterized by difficult expiration. Either more force is required to expire a given volume of air, or emptying of the lungs is slowed, or both. Spirometry (Pulmonary function tests) measures both lung volume and gas flow. Pulmonary function tests will often demonstrate partial reversibility (improvement in the FEV1 of at least 15%) after the administration of bronchodilators. The absence of improvement in pulmonary function tests after the administration of a bronchodilator does not constitute proof of irreversible airway obstruction.
Reference:
Clinical Anesthesiology 4th ed., Lange Medical Books/McGraw Hill, New York, 2006 pp574
Rita K Cydulka: Emergency Medicine 6th ed., McGraw Hill, New York, 2004 pp 468
Current Medical Diagnosis and Treatment 36th ed., Appleton and Lange, Stamford CT, 1997 pp242
Pathophysiology: The Biologic Basis for Disease in Adults and Children, Mosby, St. Louis, 1990 pp 1048, 1049

How well did you know this?
1
Not at all
2
3
4
5
Perfectly
34
Q
Which of the following medications should be avoided in a patient with a history of nasal polyposis and moderate asthma?
A. Acetaminophen (Tylenol)
B. Tramadol (Ultram)
C. Propoxyphene
D. Diclofenac
A

Answer: D
Rationale:
Aspirin induced asthma (AIA) is characterized by severe and sometimes life-threatening episodes of asthma following the use of NSAIDS (nonsteroidal anti-inflammatory drugs). Individuals with AIA suffer from chronic asthma that is often steroid dependent and associated with a significant number of individuals with chronic rhinosinusitis and nasal polyps (Sampter’s Triad). AIA usually appears in the 3rd to 4th decade of life.
Patients with AIA react to NSAIDS that inhibit cyclooxygenase enzyme 1 (COX-1). COX-1 catalyzes the formation of prostaglandins and thromboxanes from cell membrane arachidonic acid. If the COX-1 is inhibited, it may cause a shift to an alternative pathway of arachidonic acid metabolism involving 5 lipoxygenase. The products of 5 lipoxygenase include various leukotrienes that are potent inflammatory mediators. These leukotrienes can act as bronchoconstrictors. These leukotrienes may also increase mucus secretion, induce swelling of the nasal mucosa, promote airway edema and attract eosinophils into the airways. Increased leukotriene concentrations have been detected in AIA patients compared to non AIA patients both at baseline and following ASA (Aspirin) challenge.

Reference:
Jonathan P. Arm, Lung Biology in Health and Disease Vol. 115., Fatal Asthma, Marcei Dekker, 1998 pp335-345
Rita K Cydulka: Emergency Medicine 6th ed., McGraw Hill, New York, 2004 pp 475-480
Namazy J., Simon R., Current Review of Asthma., Respiratory Reactions to Anti- inflammatory Drugs, Current Medicine Inc., 2003 pp91-98
Elliot Israel MD., Up To Date, 2007 Aspirin-Induced Asthma

How well did you know this?
1
Not at all
2
3
4
5
Perfectly
35
Q
An asthmatic patient begins to complain of severe dyspnea and a cough. Auscultation of the lungs reveals wheezing. Which of the following drug classes reflects the preferred initial treatment?
A. Xanthines
B. Adrenergics
C. Anticholinergics
D. Corticosteroids
A

Answer: B
Rationale:
Beta adrenergic agents are the preferred initial rescue medication for acute bronchospasm. The ideal beta adrenergic agent would possess pure beta 2 selective activity without cardiac side effects. Older catecholamine bronchodilators such as epinephrine and isoproterenol are not beta 2 specific and have a short duration of action. Epinephrine (1:1000 or 1mg/ml 0.3mg-0.5mg SC every 20 minutes for 3 doses) may be given to patients unable to coordinate aerosolized or MDI treatments. Newer agents such as the resorcinol bronchodilators (metaproterenol, terbutaline and fenoterol) and saligenin bronchodilators (albuterol and carbuterol) have greater beta 2 specificity and a longer duration of action. Albuterol is an inhaled short-acting beta 2 agonist that can be administered as a nebulized solution or as an MDI (metered dose inhaler). Albuterol via MDI may be given at 90ug/puff with 4-8 puffs every 20 minutes up to 4 hours for the adult patient. Xanthines include medications such as theophylline. They are not considered a first line treatment for acute asthma. Xanthines are frequently reserved for those patients whose symptoms persist despite the use of an inhaled steroid and beta 2 antgonist. Therapeutic levels are often associated with gastrointestinal, cardiac, and central nervous system side effects. There is a narrow therapeutic index, toxicity may occur and be fatal. Anticholinergics such as atropine or ipratropium bromide are bronchodilators, but their usage can produce side effects such as tachycardia. Corticosteroids such as hydrocortisone and betamethasone are highly effective in the chronic management of asthma by reduction of lumen mucosa inflammation, and they may restore some beta adrenergic responsiveness, but the onset of the effect may be delayed 4-8 hours.
Reference:
Clinical Anesthesiology 4th ed., Lange Medical Books/McGraw Hill, New York, 2006 pp572-576
Rita K Cydulka: Emergency Medicine 6th ed., McGraw Hill, New York, 2004 pp 467-473

How well did you know this?
1
Not at all
2
3
4
5
Perfectly
36
Q

Auscultation of the lungs reveals the finding on the attached audio clip. What is the finding that you hear?
Sound 2: Please note that sound is not available in this format but the item content is appropriate for learning purposes.
A. Normal breath sounds
B. Rales
C. Wheezing
D. Crackles

A

Answer: C
Rationale:
The breath sound in the clip is an expiratory wheeze. Wheezing is caused by high-velocity air flow through a narrowed or obstructed airway. The longer the wheeze and the higher the pitch, the worse the obstruction. If a wheeze is heard bilaterally, it may be caused by asthma or acute or chronic bronchitis.
Crackles are heard most often during inspiration and are characterized by discrete discontinuous sounds, each lasting a few milliseconds. Crackles may be fine and high pitched or coarse and low pitched. They are caused by the disruptive passage of air through the small airways in the respiratory tree. Rales is another term for crackles. Crackles are due to abnormalities of the lungs (pneumonia, fibrosis, early congestive heart failure), or of the airways (bronchitis, bronchiectasis).
Reference:
Mosby’s Guide to Physical Examination, 6th ed. Seidel et al, 2006, p 386-387
Audio clip used with permission from Dr. Samuel Louie, University of California-Davis. Other examples of respiratory sounds can be heard at http://medocs.ucdavis.edu/IMD/420C/sounds/lngsound.htm

How well did you know this?
1
Not at all
2
3
4
5
Perfectly
37
Q
A 25 year-old male is undergoing general anesthesia for treatment of a mandible fracture. The anesthesiologist notes difficulty ventilating the patient and obtains the attached radiograph. What treatment is appropriate for the patient's condition?
A. Placement of a needle on the right
B. An albuterol nebulizer treatment
C. Antibiotics
D. Placement of a chest tube on the left
A

Answer: A
Rationale:
A tension pneumothorax is the accumulation of air in the pleural space, creating positive pressure. Because tension pneumothorax can cause hemodynamic compromise, this medical emergency requires immediate decompression. In addition to resistance to ventilation, the blood pressure decreases and central venous pressure increases. The increased pressure should be relieved immediately with needle thoracostomy, which is performed by inserting a large-bore (16 or 18-gauge) needle through the second or third intercostal space anteriorly on the involved side. A chest tube is also appropriate treatment for a tension pneumothorax, however in this case it would be on the wrong side of the patient.
An albuterol nebulizer treatment would be used for acute asthma attack. An exacerbation of asthma under anesthesia can cause difficulty with mechanical ventilation. Antibiotics would be prescribed for pneumonia. The onset of pneumonia would not typically cause acute difficulty with ventilation.
Reference:
Marx: Rosen’s Emergency Medicine: Concepts and Clinical Practice, 6th ed, Chapter 42
Roberts: Clinical Procedures in Emergency Medicine, 4th ed. Saunders, 2004, Chapter 10. Radiograph from Dr. Brian Mullan, MD, University of Iowa Hospitals and Clinics

How well did you know this?
1
Not at all
2
3
4
5
Perfectly
38
Q

A 32 year-old female presents with a chief complaint of weakness with chewing. Her symptoms are worse during evening meals. Physical examination shows diplopia and lid ptosis. If her clinical signs improved with the administration of edrophonium which of the following would you suspect?
A. Mutation in the dystrophin gene
B. Antibodies to the presynaptic voltage-gated calcium channels
C. Antibodies to sarcoglycan proteins
D. Antibodies against the postsynaptic nicotinic acetylcholine receptor

A

Answer: D
Rationale:
The patient is demonstrating classic findings associated with myasthenia gravis. This disease process is a result of autoantibodies to the postsynaptic nicotinic acetylcholine receptor. Its prevalence is about 1 in 7500 and although it affects all age groups, it is most common in males in their fifties and sixties, and females in their twenties and thirties. Cardinal features include weakness and fatigability of muscles which is progressive and may improve with rest. Cranial muscle weakness usually involves the eyelids and extraocular muscles early on resulting in diplopia and ptosis. Weakness with chewing may result after a prolonged effort, and nasal speech as well as difficulty swallowing may also result from palatal and pharyngeal muscle weakness. Limb weakness is proximal and may be asymmetric. Deep tendon reflexes are preserved. Occasionally, respiratory muscle weakness may develop requiring ventilatory assistance; this is known as a myasthenic crisis. Foil A is associated with Duchenne Muscular Dystrophy. Foil B represents the pathophysiology of Lambert-Eaton Myasthenic Syndrome. Foil C is the etiology of Limb- Girdle muscular dystrophy.
Reference:
Fauci et al., Harrison’s Principles of Internal Medicine 14th ed., McGraw-Hill, 1998, pp. 2469-2472.
Goetz: Textbook of Clinical Neurology, 2nd ed., Copyright © 2003 Saunders, An Imprint of Elsevier

How well did you know this?
1
Not at all
2
3
4
5
Perfectly
39
Q
A 67 year-old gentleman presents to your office with xerostomia and keratoconjunctivitis sicca. He has a long history of smoking and recently diagnosed lung cancer. You note intermittent eyelid ptosis, and decreased deep tendon reflexes that improve briefly with exertion, but with sustained activity worsen. Which disorder do you suspect?
A. Hyperthyroidism
B. Myasthenia gravis
C. Oculopharyngeal dystrophy
D. Lambert-Eaton myasthenic syndrome
A

Answer: D
Rationale:
Lambert-Eaton myasthenic syndrome is a disorder of the presynaptic neuromuscular junction. It causes symptoms that resemble closely those of myasthenia gravis. However, it is typically associated with a malignancy, most commonly small cell cancer of the lung. This is thought to be caused by autoantibodies to the calcium channels at motor nerve terminals which impairs the release of acetylcholine. Treatment of the disorder involves immunosuppression often with steroids, as well as plasmapheresis, which is often performed preoperatively, along with cessation of the anticholinesterase medications. A differential diagnosis includes: hyperthyroidism, botulism, intracranial lesions, neurasthenia, and progressive external ophthalmoplegia.
Reference:
Fauci et al., Harrison’s Principles of Internal Medicine 14th ed., McGraw-Hill, 1998, pp. 2469-2472.
Ferri: Ferri’s Clinical Advisor 2007: Instant Diagnosis and Treatment, 9th ed., Copyright © 2007 Mosby, An Imprint of Elsevier

How well did you know this?
1
Not at all
2
3
4
5
Perfectly
40
Q
A 5 year-old male presents to your office with his mother. In the dental chair, the boy pushes his hands against his knees to elevate his upper torso into an upright position. You also notice that his calf muscles are rather large when compared with his thighs, and that he is intellectually impaired. What is the disease process is most likely affecting this young person?
A. Myasthenia gravis
B. Dermatomyositis
C. Polymyositis
D. Duchenne muscular dystrophy
A

Answer: D
Rationale:
The boy performs a classic Gowers’ maneuver when elevating himself to an erect position, which is a giveaway for the diagnosis of Duchenne muscular dystrophy. This is a hereditary myopathy that is also known as pseudohypertrophic muscular dystrophy. It is an X-linked recessive disorder with an incidence of 30 per 100,000 males. While present at birth, it usually becomes apparent between the ages of 3 and 5. The boys have difficulty keeping up with physical activities with their peers, and have a progressive muscle weakness. The enlarged calves are also a classic physical finding and while inappropriately termed pseudohypertrophy, it is actually a result of fibrofatty tissue replacing the muscle in the enlarged calves. These children typically are unable to walk by age 12, and also suffer from scoliosis and contractures. They commonly suffer from intellectual impairment and have an IQ approximately one standard deviation below their peers. Cardiomyopathy is present in most of these patients, though death is usually due to pulmonary infections and an increased incidence of aspiration pneumonia.
Reference:
Fauci et al., Harrison’s Principles of Internal Medicine 14th ed., McGraw-Hill, 1998, pp. 2473-2475.
Behrman: Nelson Textbook of Pediatrics, 17th ed., Copyright © 2004 Saunders, An Imprint of Elsevier, Chapter 600.

How well did you know this?
1
Not at all
2
3
4
5
Perfectly
41
Q
What finding is commonly associated present with Sturge-Weber Syndrome?
A. Seizures
B. Lip pits
C. CafÈ-au-lait spots on trunk
D. Pronounced mandibular hypoplasia
A

Answer: A
Rationale:
The classic clinical manifestations of Sturge-Weber Syndrome are facial vascular nevi (port-wine stain), epilepsy, cognitive deficits, and hemiparesis or hemiplegia, hemianopia, glaucoma. Patients with Sturge-Weber syndrome may actually have enlargement of the associated lip, maxilla, or mandible. Cerebral angiography of these patients reveals capillary, venous and arteriovenous anomalies of the leptomeninges. The anomalous circulation is responsible for the progressive degeneration and atrophy of the cerebral hemispheres. Extensive leptomeningeal abnormalities can cause seizures, contralateral hemiplegia, and delayed motor and cognitive skills.
CafÈ-au-lait spots on a patient’s trunk are commonly associated with McCune-Albright Syndrome. Pronounced mandibular hypoplasia can be associated with such conditions as Pierre-Robin Sequence or Treacher-Collins Syndrome but is not associated with Sturge- Weber Syndrome.
Reference:
Goldman L, Ausiello D: Cecil Textbook of Medicine 22nd ed., Saunders, Philadelphia, 2003 pp 2363
Cummings: Otolaryngology: Head & Neck Surgery, 4th ed., Copyright © 2005 Mosby, Inc, Chapter 174 ñ Vascular tumors and malformations of the head and neck, capillary malformation.

How well did you know this?
1
Not at all
2
3
4
5
Perfectly
42
Q
A 24 year-old patient with a panfacial fracture acutely develops speech difficulties and confusion. You have been called to the surgical intensive care unit to examine him for a newly discovered maxillary fracture. While talking with the patient, he acutely develops difficulty speaking despite remaining fully conscious. His vital signs remain stable, and he recovers fully after several minutes. What is the most likely explanation for his acute speech difficulties and confusion?
A. Transient ischemic attack
B. Hypoglycemia
C. Seizure
D. Narcolepsy
A

Answer: C
Rationale:
Posttraumatic seizures can occur as an acute result of blunt or penetrating head trauma. Immediate posttraumatic seizures occur with 24 hours of injury, and are more common in children than in adults. Within the first year after significant head trauma, the incidence of seizures is at least 12 times that of the general population. The incidence of seizures after injury with neurologic deficit without dural violation is 7% to 39%. When the dura is disrupted, the incidence is 20 ñ 57%. Most patients with pos-traumatic epilepsy have partial seizures that can affect sensory, motor or autonomic areas of the brain. If the speech centers are involved in the ictal discharge, speech can be disturbed or arrested. These seizures are usually short lived, and the patients recover function quickly.
The other answers can be associated with speech difficulties but are not necessarily related to head trauma and have other related symptoms. A transient ischemic attack could involve speech disturbance, but the patient is in the wrong age group and did not have any other symptoms of a TIA such as decreased consciousness, nausea, vomiting, visual disturbance, or hemiplegia. Most patients who suffer TIAs do not fully recover in minutes. Hypoglycemic attacks can cause speech alteration, but they are associated with other symptoms such as confusion, loss of consciousness, nausea, and vomiting. These attacks do not usually resolve spontaneously, but rather require intervention. Narcolepsy is associated with decreased consciousness or sudden onset of a sleep state.
Reference:
Goldman L, Ausiello D: Cecil Textbook of Medicine 22nd ed., Saunders, Philadelphia, 2003 pp 2257-2269.
Marx: Rosen’s Emergency Medicine: Concepts and Clinical Practice, 6th ed, 2006; Chapter 100 ñ Seizures ñ Seizures caused by trauma.

How well did you know this?
1
Not at all
2
3
4
5
Perfectly
43
Q
A 25 year-old female with multiple sclerosis is undergoing surgery in the operating room under general anesthesia. Which of the following neuromuscular blocking drugs is contraindicated in this individual?
A. Vecuronium
B. Cisatracurium
C. Rocuronium
D. Succinylcholine
A

Answer: D
Rationale:
Neuromuscular blocking drugs should be used judiciously. Succinylcholine poses the risk of hyperkalemia due to the release of intracellular potassium. Hyperkalemia after depolarizing neuromuscular block represents a risk to all patients with muscle denervation pathology, and may lead to cardiac arrest. Patients with lesions involving motor nuclei, as evidenced by flaccidity, spasticity or hyperreflexia, are at risk for hyperkalemia. Upregulation of acetylcholine receptors even by the fourth day after the onset of symptoms leads to sensitivity to succinylcholine in multiple sclerosis patients. Patients remain at risk for hyperkalemia for months or years and succinylcholine is best avoided. The use of nondepolarizing muscle relaxants can also pose difficulties. In addition to their multiple interactions with medications taken by multiple sclerosis patients, they have a variable pharmacodynamic effect. Denervation with upregulation of acetylcholine receptors may increase resistance to nondepolarizing neuromuscular blocking drugs. This resistance may outlast the patients’ sensitivity to succinylcholine. On the other hand, muscle weakness and decreased muscle mass may be associated with increased sensitivity. Careful titration, continual monitoring and use of the lowest necessary dose, represent prudent management principles. The nerve twitch should be monitored, if possible, on an unaffected extremity so that factors such as increased resistance to neuromuscular blocking drugs will not lead to unintended overdoses.
Reference:
Dorotta IR, Schubert A. Multiple sclerosis and anesthetic implications. Current Opinion in Anaesthesiology 2002, 15:365-370.
Miller’s Anesthesia, 6th ed. 2005, Churchill Livingston, Chapter 13 ñ Pharmacology of Muscle Relaxants and their antagonists ñ demyelinating diseases.

How well did you know this?
1
Not at all
2
3
4
5
Perfectly
44
Q

A definitive diagnosis of Alzheimerís disease can only be made by which of the following tests or exams?
A. Psychiatric examination
B. Neurologic examination
C. CNS imaging studies (CT scan, MRI, and PET scan) D. Examination of brain tissue at autopsy

A

Answer: D
Rationale:
The dementing illness of Alzheimer’s disease is insidious in its onset, very gradual in its progression, and ultimately fatal. It manifests itself as changes in memory and daily functioning and is characterized by anterograde amnesia, cognitive decline and dementia which lead to the patient’s inability to participate in self-care, disruptions in speech, and swallowing abnormalities. Seven stages of disease progression have been developed to better define the progression nature of the cognitive deterioration. A presumptive clinical diagnosis of Alzheimer’s disease is obtained by identifying the patient’s clinical symptoms and comparing them with a set of known criteria. Patients generally undergo complete physical, neurologic and psychiatric examinations. An attempt is made to rule out treatable illnesses such as Parkinson’s disease, drug toxicity, metabolic diseases, dietary deficiency, cerebrovascular accident, and CNS infective diseases (e.g. syphilis or HIV). Common diagnostic tools include Mini Mental State Examination, CBC, serum electrolytes, liver function tests, cholesterol, serum lipids, vitamin B12 and folate, thyroid function tests, urinalysis, chest x-ray, ECG, EEG, and CNS imaging studies. Though each of these tests, when taken as a whole, can lead identification of Alzheimer’s disease through a process of elimination, the diagnosis can only be made with certainty at time of the patient’s death when the brain tissue is examined for intra -cellular fl-amyloid plaques, extracellular fl- amyloid and intracellular neurofibrillary tangles of tau protein in the entorhinal cortex, hippocampus, basal forebrain, amygdale and parietotemporal cortex ñ areas related to memory, learning, language, and emotional behavior.
Reference:
Braak H, Braak E. Neuropathological staging of Alzheimer-related changes. Acta Neuropathol 1991; 82:239.
Braak H, Braak E. Evolution of neuronal changes in the course of Alzheimerís disease. J Neural Transm Suppl 1998; 53:127.
Haaroutunian V, Perl DP, Purohid DP, et al. Regional distribution of neuritic plaques in the nondemented elderly and subjects with very mild Alzheimer disease. Arch Neurol 1998; 55:1185

How well did you know this?
1
Not at all
2
3
4
5
Perfectly
45
Q
A 65 year-old male underwent general anesthesia for mandible fracture repair one day ago. His family notes weakness of his right arm and slurring of speech over the last 30 minutes. Which of the following studies should be initially performed?
A. Lumbar puncture
B. Carotid arteriogram
C. Carotid doppler
D. Noncontrast head CT
A

Answer: D
Rationale:
Brain imaging remains a required component of the emergency assessment of patients with suspected stroke. Both CT and MRI are options for imaging the brain, but for most cases and at most institutions, CT remains the most practical initial brain imaging test. Head CT will determine if the cause of arm weakness and slurred speech is a CVA and whether it is hemorrhagic or ischemic. Ischemic strokes comprise approximately 80% of strokes. Thrombolytic therapy is ideally initiated within 3 hours of the event to decrease the long- term disability of the individual.
Most patients with stroke do not need an examination of cerebrospinal fluid. The yield of brain imaging is very high for detection of intracranial hemorrhage. The clinical course of subarachnoid hemorrhage or acute central nervous system infections usually is distinct from that of ischemic stroke. Examination of the cerebrospinal fluid may be indicated for evaluation of a patient with a stroke that may be secondary to an infectious illness.
In addition to CT and MR angiography, transcranial Doppler ultrasonography, carotid duplex sonography, and catheter angiography have been used to detect intracranial or extracranial vessel abnormalities, but head CT would be the initial test for imaging.
Reference:
Kokko, J., Stein, S., The Emory University Comprehensive Board Review in Internal Medicine. McGraw-Hill. New York, 2000, p711-712.
Adams H, et al. Guidelines for the Early Management of Adults with Ischemic Stroke; Stroke 2007; 38: 1655.

How well did you know this?
1
Not at all
2
3
4
5
Perfectly
46
Q
A 32 year-old patient with rheumatoid arthritis presents for treatment of partially erupted tooth #32 with pericoronitis. The patient takes methotrexate. Which medication should be avoided in the post-operative management of this individual?
A. Hydrocodone
B. Codeine
C. Tylenol
D. Ibuprofen
A

Answer: D
Rationale:
Methotrexate is used for the management of rheumatoid arthritis. The mechanism of action of methotrexate is that it is a folate antimetabolite that inhibits DNA synthesis. Methotrexate irreversibly binds to dihydrofolate reductase, resulting in inhibition of or purine and thymidylic acid synthesis.
Methotrexate has side-effects which include bone marrow suppression, aplastic anemia, and GI toxicity with concomitant administration of NSAIDs. Penicillins may increase methotrexate concentrations due to a reduction in renal tubular secretion. Tylenol and opioids may be administered to this patient.
Severe, sometimes fatal, toxicity (including hematologic and GI toxicity) has occurred following administration of a NSAID (e.g., indomethacin, ketoprofen) concomitantly with methotrexate (particularly with high-dose therapy) in patients with various malignant neoplasms, psoriasis, or rheumatoid arthritis. The toxicity was associated with elevated and prolonged serum concentrations of methotrexate. The exact mechanism of the interaction remains to be established, but it has been suggested that NSAIDs may inhibit renal elimination of methotrexate, possibly by decreasing renal perfusion via inhibition of renal prostaglandin synthesis or by competing for renal elimination.
NSAIDs should be avoided in patients receiving relatively high dosages of methotrexate (e.g., those used in the treatment of neoplastic disease). The risk of concomitant low-dose, intermittent (e.g., 5-15 mg weekly) methotrexate therapy and NSAIDs has not been fully elucidated, but the drugs have been used concomitantly in many patients receiving methotrexate for the management of rheumatoid arthritis. However, in clinical studies in which the drugs were used concomitantly, the patients often were monitored closely and were receiving relatively stable dosages of NSAIDs; in addition, those with conditions that might predispose to methotrexate toxicity generally were excluded from the studies. NSAIDs should be used with caution in patients receiving low-dose methotrexate regimens such as those employed in the management of rheumatoid arthritis, and the possibility of increased and prolonged serum methotrexate concentrations and resultant toxicity should be considered. Although intermittent regimens also are used in the management of psoriasis, methotrexate dosages in such regimens usually are higher than those used in the management of rheumatoid arthritis and therefore are more likely to result in toxicity during concomitant NSAID therapy; serious toxicity, including at least one death, has been reported in several patients with psoriasis receiving combined therapy with the drugs. Further study is needed to evaluate the interaction between NSAIDs and methotrexate.
Reference:
AHFS Drug Information, McEvoy, American Society of Health-System Pharmacists, Inc., 2007, Methotrexate

How well did you know this?
1
Not at all
2
3
4
5
Perfectly
47
Q
Eye examination of a trauma patient in the emergency department reveals one pupil to be fixed and dilated, with that same eye deviated laterally and downward, with ptosis of the eyelid. Which of the following cranial nerves is injured?
A. Optic (II)
B. Oculomotor (III)
C. Trochlear (IV)
D. Abducens (VI)
A

Answer: B
Rationale:
The oculomotor nerve has been injured in this case. This nerve supplies motor function to all of the extraocular muscles, except the lateral rectus (supplied by the abducens nerve) and the superior oblique (supplied by the trochlear nerve). Thus the only nerves functioning have caused a downward and lateral gaze in the affected eye. The oculomotor nerve also supplies motor function to the levator palpebrae superioris. Loss of this function has caused ptosis of the eyelid. Parasympathetic fibers carried by cranial nerve III from the ciliary ganglion to the pupillary sphincter cause pupillary constriction. Injury to these fibers causes dilation of the affected pupil.
Reference:
Seidel et al: Mosby’s Guide to Physical Examination, 6th ed., Mosby Elsevier, St. Louis, MO; 2006; p.308.
Romanes: Cunningham’s Manual of Practical Anatomy, Oxford Medical Publications, Oxford; 2006; p.113.

How well did you know this?
1
Not at all
2
3
4
5
Perfectly
48
Q
The mechanism of action of diazepam (Valium) when treating status epilepticus occurs via a GABA mediated inhibition of what ion?
A. Calcium
B. Potassium
C. Sodium
D. Chloride
A

Answer: D
Rationale:
During seizures, extracellular potassium and intracellular calcium concentrations increase and contribute to the overall excitability of the epileptic neuronal aggregate. During the seizure itself, neurons are tonically depolarized and fire continuously in a sustained, high- frequency discharge (corresponding to the tonic phase of the seizure). The seizure ends as phasic repolarizations interrupt the continuous firing pattern (the cellular correlate of the clinical clonic phase) and gradually restore membrane potentials to normal or to a temporary hyperpolarized state (postictal depression).The benzodiazepines and barbiturates exert their anticonvulsant effect by enhancing postsynaptic γ-aminobutyric acid (GABA)- mediated inhibition through an effect on the chloride ionophore. Phenytoin and carbamazepine are effective anticonvulsants because they produce a use-dependent block of sodium channels, limiting the capability of neurons to fire at high-frequency rates.
Reference:
Goldman: Cecil Textbook of Medicine, 22nd ed.; 2004, WB Saunders, Chapter 426, Status Epilepticus
Drug Information Handbook for Dentistry, 12th ed. Wynn, Meiller, Crossley. Lexi-comp, 2006, p. 455.

How well did you know this?
1
Not at all
2
3
4
5
Perfectly
49
Q
Hornerís syndrome results from damage to which nerve fibers?
A. Sympathetic fibers to the pupil
B. Parasympathetic fibers to the pupil
C. Optic nerve
D. Oculomotor nerve
A

Answer: A
Rationale:
A unilateral small pupil is commonly due to underactivity of the ipsilateral sympathetic pathways. Miosis is commonly associated with ptosis (lip droop) due to sympathetic denervation of the tarsal muscle and facial anhidrosis (loss of sweating). This combination is known as Horner’s syndrome. It may be due to a lesion of the hypothalamus, brain stem, cervical spinal cord, or sympathetic fibers to the pupil. Horner’s syndrome may be the first sign of an apical lung tumor (Pancoast tumor) or may occur in diseases affecting the carotid artery. The Pancoast tumor causes compression on the spinal cord at C8-T1. The sympathetic fibers to the superior cervical ganglion arise from the T1 level.
Reference:
Andeoli, Carpenter, et al. Cecil Essentials of Medicine, 7th ed. Saunders Elsevier, Philadelphia, 2007 p. 1080
Goetz: Textbook of Clinical Neurology, 2nd ed. Saunders Elsevier 2003, Chapter 21 ñ Autonomic nervous system < Pupils < Horner’s syndrome, and Chapter 15 ñ Strength and Reflexes > Lower Motor Neuron Syndromes

How well did you know this?
1
Not at all
2
3
4
5
Perfectly
50
Q
Which of the following are the vitamin-K dependent clotting factors?
A. III, VII, IX, X
B. II, VII, IX, X
C. III, VIII, IX, XII
D. II, VIII, IX, XII
A

Answer: B
Rationale:
There are 5 factors produced in the liver: II, V, VII, IX, and X. Of these, four are vitamin- K dependent II, VII, IX, and X. Vitamin K is required for gamma-carboxylation of the factors. Virtually all clotting factors are produced in the liver. Factor VIII is made in several tissues, including the liver as well as the glomerular and tubular epithelial cells in the kidney. Warfarin interferes with hepatic synthesis of vitamin K-dependent clotting factors. The PT (Prothrombin time)/INR (International normalized ratio) are both measures of the extrinsic pathway in the clotting cascade.
Reference:
Abubaker AO: Oral and Maxillofacial Surgery Secrets. Hanley & Belfus, Inc., Philadelphia, PA, 2001; Page 130
Andeoli, Carpenter, et al. Cecil Essentials of Medicine, 7th ed. Saunders Elsevier, Philadelphia, 2007 p. 532-538
Tierney LM: 2005 Curent Medical Diagnosis and Treatment. McGraw-Hill, Inc, New York, New York; 2005; Page 656

How well did you know this?
1
Not at all
2
3
4
5
Perfectly
51
Q
What is the etiology of thrombocytopenia associated with liver disease?
A. Reduced urea synthesis
B. Functional renal failure
C. Splenomegaly
D. Ascites
A

Answer: C
Rationale:
Portal hypertension is a common sequelae of chronic liver failure. Vascular congestion within the portal system leads to splenomegaly. An enlarged spleen can sequester red blood cells and platelets. Sequestration of platelets by the spleen will lead to thrombocytopenia.
Reduced urea synthesis does not lead to thrombocytopenia. Functional renal failure, also known as hepatorenal syndrome is a late complication of cirrhosis. It occurs in the presence of liver failure without intrinsic kidney abnormalities. Ascites is an accumulation of excess fluid in the abdomen and is caused by cirrhosis in at lest 80% of patients. Complications of ascites include spontaneous bacterial peritonitis, hydrothorax (passage of ascitic fluid into the pleural space) and abdominal wall hernias.
Reference:
Oral & Maxillofacial Surgery Knowledge Update. Vol 1, Part II, 1995, Patient Evaluation, p. 39.
Mercer KW, Gail Macik B, Williams ME. Hematologic disorders in critically ill patients. Semin Respir Crit Care Med. 2006 Jun; 27:286-96.
Goldman: Cecil Textbook of Medicine, 22nd ed, Chapter 157, Cirrhosis and its sequelae < Complications of Cirrhosis.

52
Q
Which of the following is a component of the modified Child-Pugh classification?
A. Albumin
B. Lactate dehydrogenase
C. Partial thromboplastin time
D. Ammonia
A

Answer: A
Rationale:
Modified Child-Pugh classification was designed to indicate the severity of cirrhosis in patients with chronic liver disease. The components of the classification include: albumin, bilirubin, PT, INR, ascites, and encephalopathy. PTT is not a component of the Child-Pugh classification. PTT is used to measure the intrinsic pathway. PT is used as a measure of the extrinsic pathway. Ammonia is detoxified into urea by the liver. Increased levels of ammonia are thought to contribute to hepatic encephalopathy. Lactate dehydrogenase levels can be elevated in hepatitis.
Modified Child-Pugh scoring system Modified Child-Pugh Score * Parameters 1 2 3
Albumin (g/dL) >3.5 1.8ñ3.5 6
Internationalnormalizedratio2.3
Bilirubin (mg/dL) Ü 3
Ascites Absent Slight-moderate Tense
Encephalopathy None Grade IñII Grade IIIñIV
From Pugh RNH, Murray-Lyon IM, Dawson JL, et al: Transection of oesophagus for bleeding of oesophageal varices. Br J Surg 60:646ñ649, 1973.
* ClassA=5to6points,B=7to9points,andC=10to15points.
Reference:
Martins A, Cortez-Pinto H, Marques-Vidal P, Mendes N, et al. Treatment and prognostic factors in patients with hepatocellular carcinoma. Liver Int. 2006 Aug; 26(6):680-7.
Miller: Miller’s Anesthesia, 6th ed, Chapter 55 ñ Anesthesia and the Hepatobiliary System < Cirrhosis as a perioperative risk factor.

53
Q
Detection of only Hepatitis B surface antibody (Anti-HBsAg) in the serum is associated with which of the following?
A. Chronic infection
B. Acute infection
C. Immunity
D. Initial cellular response
A

Answer: C
Rationale:
Hepatitis B surface antigen encodes the viral envelope of the virus. Anti-HBsAg is the antibody formed against this antigen and its presence confers immunity when the titer is > 10mIU/ml. The following table outlines the differences in immunity, acute, and chronic infectious states of Hepatitis B in regards to the serologic markers.


54
Q

First line treatment for primary peptic ulcer disease would be:
A. H2 blockers and antacids.
B. mucosal protectants and antibiotics.
C. proton pump inhibitors and H2 blockers.
D. proton pump inhibitor and antibiotics.

A

Answer: D
Rationale:
Peptic ulcers are called primary when they occur without any predisposing factors such as acute medical illness trauma, use of NSAIDS, alcohol, or smoking. Helicobacter pylori is recognized as an important factor in the pathogenesis of primary peptic ulcer disease. Inhibitors of gastric acid secretion and antimicrobial agents are recommended. This combination therapy is first line treatment because antisecretory drugs provide rapid relief of pain and accelerate healing; while antibiotics eradicate Helicobacter pylori.
In the United States, perhaps 80% of patients who have duodenal ulcers are infected with the organism, as are more than 60% of those with gastric ulcers.
Reference:
Perioperative Management of the Oral and Maxillofacial Surgery Patient, Part II, Oral and Maxillofacial Surgery Clinics of North America, Vol. 18, No. 2, May 2006,
pp. 245-246
Dental Management of the Medically Compromised Patient, 6th Ed., 2002, Little, J.W., Falace, D.A., Miller, C.S., Rhodus, N.L., pp. 192-193
Feldman: Sleisenger & Fordtran’s Gastrointestinal and Liver Disease, 8th ed., Saunders, 2006, p.1094.

55
Q
Barrettís metaplasia is a complication associated with which of the following?
A. Chronic ulcerative colitis
B. Peptic ulcer
C. Gastroesophageal reflux
D. Crohnís disease
A

Answer: C
Rationale:
Barrett’s esophagus is not uncommon, and is found in approximately 6% to 12% of patients undergoing endoscopy for symptomatic GERD and in 1% to 2% of unselected patients undergoing endoscopy.
Barrett’s esophagus occurs when the squamous epithelium of the esophagus is replaced by metaplastic columnar epithelium. It develops due to chronic reflux-induced injury to the esophageal squamous epithelium. These patients often have severe GERD, usually with chronic reflux symptoms for more than 10 years, poor esophageal motility, large hiatal hernias, and extensive acid and bile reflux. Patients with long-segment Barrett’s esophagus have an estimated 30 to 125 times increased risk of developing esophageal cancer compared with the general population. Epidemiologic data suggest that the mean interval from developing Barrett’s esophagus to evolution to cancer may be 20 to 30 years.
The appropriate surveillance interval for patients with Barrett’s esophagus has not been studied prospectively. However, current programs, such as those proposed by the American College of Gastroenterology, are based on the grade of dysplasia. Individuals with high- grade dysplasia may undergo endoscopy every 3 months, while those with low grade dysplasia may be followed yearly.
Reference:
Current Medical Diagnosis and Treatment, 42nd Ed., 2003, Tierney, L.M., McPhee, S.J., Papadakis, M.A., pp. 552-553
Feldman: Sleisenger and Fordtran’s Gastrointestinal and Liver Disease, 8th ed., Saunders, 2006, 921-929.

56
Q

Which of the following differentiates type I from type II diabetes mellitus?
A. Ketoacidosis does not occur in type II DM
B. There is a stronger familial link with type I DM than with type II DM
C. Insulin use is only indicated in the management of type I DM
D. Diabetic neuropathy is primarily a complication of type II DM

A

Answer: A
Rationale:
Since insulin production continues to be produced in individuals with type II DM, ketoacidosis does not occur though insulin may ultimately be required to control hyperglycemia. Genetic tendencies towards developing diabetes are associated with both types, but the linkage is more prominent in type II DM where there is concordance rate of greater than 90% between identical twins. Those with type II DM often progress to the need for exogenous insulin to control hyperglycemia. Diabetic neuropathy is a frequent complication of both types of DM.
Diabetic ketoacidosis (DKA) is a syndrome in which insulin deficiency and glucagon excess combine to produce a hyperglycemic, dehydrated, acidotic patient with profound electrolyte imbalance. Glucose in the renal tubules draws water, sodium, potassium, magnesium, calcium, phosphorus, and other ions from the circulation into the urine. This osmotic diuresis combined with poor intake and vomiting produces the profound dehydration and electrolyte imbalance associated with DKA.
In uncontrolled type II DM, the patient develops a hyperosmolar nonketotic state. Decreased insulin action results in glycogenolysis, gluconeogenesis, and decreased peripheral uptake of glucose. The hyperglycemia pulls fluid from the intracellular space into the extracellular space, transiently maintaining adequate perfusion. Soon, however, this fluid is lost in a profound osmotic diuresis, limited finally by hypotension and a subsequent drop in the glomerular filtration rate (GFR). The urine is extremely hypotonic, with urine sodium concentration between 50 and 70 mEq/L, compared with 140 mEq/L in extracellular fluid. This hypotonic diuresis produces profound dehydration, leading to hyperglycemia, hypernatremia, and associated hypertonicity.
Reference:
Baker, Maurer, Wauman; Perioperative management of diabetes mellitus, Oral and Maxillofacial Clinics of North America: 10:3:363-371; W.B.Saunders, Philadelphia, 1998
Powers: C.323, Diabetes mellitus, Harrison’s Principles of Internal Medicine, 16th Edit., p.2169-2185; McGraw-Hill, New York, 2005

57
Q
Cushingís syndrome is most commonly due to which of the following?
A. A functioning pituitary adenoma
B. Adrenal carcinoma
C. Exogenous steroid administration
D. Adrenal hyperplasia
A

Answer: A
Rationale:
Cushing’s syndrome is due to chronic exogenous or endogenous glucocorticoid excess.
The most common cause of Cushing’s syndrome is a functioning pituitary adenoma (termed Cushing’s disease). Approximately 80% of ACTH-dependent cases are due to this etiology. Cushing’s Syndrome resulting from endogenous sources may be ACTH- dependent or ACTH-independent. Ectopic ACTH syndrome accounts for about 10% of cases. Examples of ACTH-independent endogenous sources are: pituitary adenoma, small- cell lung carcinoma, bronchial carcinoma. Cushing’s Syndrome can also be due to exogenous iatrogenic steroid use.
Signs and symptoms of Cushing’s syndrome include: centripetal obesity, reproductive dysfunction, psychiatric abnormalities, osteoporotic changes leading to pathologic fractures, thinning of the skin, myopathy and bruising, hypertension, infections are common, glucose intolerance with diabetes, electrolyte imbalance, water retention, and increased intraocular pressure and exophthalmos.
Reference:
Andreoli T: Cecil Essentials of Medicine., Mosby, Philadelphia, 2001 pp569-570
Henderson K, Baranski T, Bickel P: The Washington Manual Endocrinology Subspecialty Consult., Lippincott Williams & Wilkins, Philadelphia, 2005 pp76-77
Larsen: Williams Textbook of Endocrinology, 10th ed. 2003, p. 508-525.

58
Q

The physical findings in a patient with untreated Addisonís disease include:
A. skin striae.
B. hyperpigmentation of oral mucosa and skin.
C. centripetal obesity.
D. acne.

A

Answer: B
Rationale:
Primary adrenocortical insufficiency is known as Addison’s disease. The cause is most often the result of autoimmune destruction of the adrenal glands. The loss of ACTH secretion seen in secondary adrenocortical insufficiency is due to pituitary suppression resulting from long term use of exogenous glucocorticoids. Skin striae, centripetal obesity, and adult onset acne are seen in patients diagnosed with Cushing’s Syndrome (adrenocortical excess). Hyperpigmentation of oral mucosa and skin is seen in patients with primary adrenocortical deficiency (Addison’s disease) because ACTH continues to be produced by the normally functioning pituitary gland. ACTH directly activates melanocortin receptors resulting in pigmentation of mucosa and skin. Other signs and symptoms frequently seen in patient’s with adrenocortical insufficiency (primary or secondary) include; nausea, vomiting, anorexia, weight loss, and fatigue.
Adrenal crisis is characterized by: dehydration, hypotension or shock, nausea/vomiting, severe abdominal pain, unexplained hypoglycemia and fever, hyperpigmentation, electrolyte abnormalities (hyponatremia, hyperkalemia, azotemia, hypercalcemia), and hypothyroidism.
Adrenal crisis is precipitated by stress. Acute adrenal insufficiency is a life-threatening emergency, and treatment should not be delayed while waiting for definitive proof of diagnosis. However, in addition to measurement of plasma electrolytes and blood glucose, appropriate samples for ACTH and cortisol should be taken before giving corticosteroid therapy. If the patient is not critically ill, an acute ACTH stimulation test can be performed.
Emergency measures for treatment of acute adrenal insufficiency include the establishment of IV access, drawing blood for serum electrolytes and glucose, as well as measurement of plasma cortisol and ACTH. An infusion of 0.9% normal saline, with or without 5% dextrose is started. Hydrocortisone 100 mg is administered and repeated every 6 hours.
Prevention of an adrenal crisis in a minor illness or stressful situation is to increase the glucocorticoid dose two- to three-fold for the few days of the illness. For minor procedures under local anesthesia, no extra supplementation is needed. For moderately stressful
procedures, a single 100 mg intravenous dose of hydrocortisone just before the procedure is given.
For major surgery, give hydrocortisone 100 mg intravenously just before induction of anesthesia and continue every 8 hr for first 24 hr. Taper dose rapidly, decreasing by half per day, to maintenance level.
Reference:
Greenspan F, Gardener D: Basic & Clinical Endocrinology., Lange Medical Books/McGraw-Hill, New York, 2004 pp384-386
Henderson K, Baranski T, Bickel P: The Washington Manual Endocrinology Subspecialty Consult., Lippincott Williams & Wilkins, Philadelphia, 2005 pp61-66
Larsen: Williams Textbook of Endocrinology, 10th ed. 2003, p. 525-527.

59
Q
In the table below, which set of laboratory values would most commonly be noted in the primary hyperthyroid patient?
A T4 down T3 down TSH up
B T4 up T3 up TSH down
C T4 down T3 down TSH down
D T4 up T3 up TSH up
A

Answer: B
Rationale:
The patient with primary hyperthyroidism has elevations of both circulating T3 and T4 from thyroid gland hyperplasia, nodule, or adenoma/adenocarcinoma. This elevation of circulating hormones exerts an inhibitory influence on the release of TSH by the anterior pituitary gland, via a negative feedback mechanism. By extension, the hypothyroid patient tends to have decreased levels of circulating T3 and T4, with concomitant elevation of the TSH level. An additional test which is of use is the Resin T3 Uptake test. The RT3U test measures the level of thyroid hormone-binding proteins in the blood. It is typically elevated in the hyperthyroid patient, and decreased in the hypothyroid patient.
If, however, there is a lesion in the pituitary gland, e.g., hyperplasia or adenoma, then TSH is oversecreted. Because TSH is elevated, T3 and T4 secretion is elevated. This is secondary hyperthyroidism (secondary to pituitary disease).
In most individuals with hypothyroidism, serum TSH results are clearly elevated, but results may be inappropriately normal for the level of T4 and T3 in those with pituitary or hypothalamic disorders.

60
Q
A 9 year-old male presents with oral ulcerations and generalized erythematous gingival enlargement. A review of systems elicits a history of abdominal pain and intermittent diarrhea. An endoscopic evaluation showed ulcerations in the ileum and cecum separated by areas of normal appearing mucosa. This clinical presentation is most consistent with:
A. Crohnís disease.
B. Wegenerís granulomatosis.
C. ulcerative colitis.
D. irritable bowel syndrome.
A

Answer: A
Rationale:
Crohn’s disease is an inflammatory bowel disease that can affect young adults and present with oral lesions such as aphthous ulcers of the buccal mucosa, lips or gingiva. Generalized gingival enlargement has also been described. Wegener’s granulomatosis can also present with oral lesions but is best described as a granulomatous vasculitis affecting primarily the upper respiratory tract, lungs, and kidneys. Endoscopic findings of bowel ulcerations separated by areas of normal mucosa or ìskip lesionsî are common in Crohn’s disease and are not characteristic of ulcerative colitis. Kaposi’s sarcoma is a vascular neoplasm seen often in patients with AIDS and oral lesions appear as red-blue macules which progress to bluish nodules.

61
Q
A 25 year-old female with progressive mandibular prognathism presents with facial features, widely spaced teeth and bulbous finger tips. Which of the following is the most likely cause?
A. Hyperthyroidism
B. Pheochromocytoma
C. Chronic hypercalcemia
D. Pituitary adenoma
A

Answer: D
Rationale:
This patient has findings highly suggestive of acromegaly, the result of excess levels of circulating growth hormone (GH). GH exhibits anti-insulin actions and about 10 % of those with acromegaly develop diabetes mellitus. GH levels should be measured. A GH level greater than 10 ng/ml favors a diagnosis of acromegaly. If GH is not suppressed below 5 ng/ml one to two hours after ingestion of 100 g of glucose, the diagnosis of acromegaly is more conclusive. The cause of acromegaly is often a pituitary adenoma which secretes GH. Often the adenoma is large enough to cause distortion of the sella turcica and may be evident on lateral cephalometric radiographs. Hypercalcemia does not cause the facies described. Hyperthyroidism does not cause increased levels of GH. Pheochromocytoma has only rarely been known to express growth hormone releasing factor (GHRF) activity causing acromegaly.

62
Q
Diabetic neuropathy can directly lead to which of the following conditions?
A. Postural hypotension
B. Resting bradycardia
C. Arrhythmias
D. Cardiomyopathy
A

Answer: A
Rationale:
Diabetic neuropathy is not a single entity but a number of different syndromes, ranging from subclinical to clinical manifestations depending on the classes of nerve fibers involved.
Diabetic neuropathy can cause postural hypotension. Patients with type 2 diabetes mellitus and orthostatic hypotension are hypovolemic and have sympathoadrenal insufficiency. Both factors contribute to the pathogenesis of orthostatic hypotension. Postural hypotension in the patient with diabetic autonomic neuropathy can present a difficult management problem. Elevating the blood pressure in the standing position must be balanced against preventing hypertension in the supine position.
It does not directly cause bradycardia, arrhythmias or cardiomyopathy unless brought about indirectly by ischemic coronary athlerosclerosis. Diabetic neuropathy can also result in dysphagia, gastroparesis, constipation, and abnormal sweating.
A constellation of metabolic derangements that are frequently seen in patients with insulin resistance and type 2 diabetes are individually associated with an increased risk of cardiovascular disease. These patients have been variously designated as having syndrome X; the dysmetabolic syndrome; hypertension, obesity, nonñinsulin-dependent diabetes mellitus (NIDDM), dyslipidemia, and atherosclerotic cardiovascular disease (HONDA); or the “deadly quartet.” Myocardial infarction, stroke, and nonischemic cardiovascular disease are the cause of death in up to 80% of individuals with type 2 diabetes. Independent of other risk factors, type 2 diabetes increases the risk for cardiovascular morbidity and mortality but also provides a synergistic interaction with other risk factors such as smoking, hypertension, and dyslipidemia.

63
Q
Intense dilute diuresis following head trauma is caused by:
A. increased vasopressin production.
B. decreased vasopressin production.
C. increased renin production.
D. decreased renin production.
A

Answer: B
Rationale:
Trauma to the hypothalamus produces a decrease in vasopressin synthesis and secretion. A normal kidney will attempt to reabsorb or excrete solute-free water to preserve a normal plasma osmolality of 275 to 290 mOsm/kg. The primary hormone regulating plasma osmolality is arginine vasopressin. It is synthesized in the hypothalamus and released into the systemic circulation by means of the posterior pituitary gland. Despite wide fluctuations in water and sodium intake, the body normally can maintain serum osmolality in a narrow range (275 to 290 mOsm/kg). Osmoreceptors near the hypothalamus sense plasma osmolality and modulate vasopressin release. Vasopressin functions at the distal collecting duct of the kidney to increase water reabsorption in this otherwise relatively water- impermeable section of the nephron. In hypo-osmolar conditions for instance, vasopressin levels fall to a low basal rate to reabsorb less free water, resulting in more dilute urine.
The situation described is consistent with diabetes insipidus, where there is decreased release of vasopressin, also known as antidiuretic hormone (ADH). Patients with diabetes insipidus and inadequate thirst can rapidly become dehydrated and may experience severe hypernatremia with devastating effects on the CNS. Hypertonic encephalopathy with obtundation, coma, and seizures may be produced by brain shrinkage. A decreased volume of brain in the skull may lead to subarachnoid hemorrhage, intracerebral bleeding, or petechial hemorrhage.
Because a head trauma patient must be given fluids parenterally, some clinicians prefer to use a continuous infusion of low-dose vasopressin. The vasopressin can either be added directly to the crystalloid solution that is being administered or infused separately to maintain a constant antidiuresis while fluid intake is adjusted appropriately to any persistent polyuria and to cover insensible water loss. Doses of 0.25 to 2.7 mU/kg per hour have been described. With this method, there is a potential to produce hyponatremia, and serum sodium levels must be checked regularly.
SIADH is produced when plasma levels of arginine vasopressin are elevated at times during which the physiologic secretion of vasopressin from the posterior pituitary would normally be suppressed. Because the clinical abnormality is a decrease in the osmotic pressure of body fluids, the hallmark of SIADH is hypo-osmolality, with euvolemia. Clinical criteria for diagnosis include: Plasma osmolality 100 mOsm/kg with normal renal function, clinical euvolemia, elevated urinary sodium excretion with normal salt and water intake, absence of other causes (hypothyroidism, addisonís disease, diuretic use, pituitary ACTH insufficiency).
The pathophysiology of primary polydipsia is essentially the reverse of that in central DI: the excessive intake of water expands and slightly dilutes body fluids, suppresses ADH secretion, and dilutes the urine. The resultant increase in the rate of water excretion balances the increase in intake, and the osmolality of body water stabilizes at a new, slightly lower level that approximates the osmotic threshold for ADH secretion. Primary polydipsia can be produced by drugs that cause a dry mouth, or by any peripheral disorder causing pathologic elevations of renin or angiotensin. Primary polydipsia is often due to a severe mental illness such as schizophrenia, mania, or an obsessive-compulsive disorder, in which case it is called psychogenic polydipsia.

64
Q
An individual diagnosed with Cushingís disease would demonstrate which abnormality?
A. Hypovolemia
B. Hyperkalemia
C. Hyperglycemia
D. Hypotension
A

Answer: C
Rationale:
Cushing’s disease involves glucocorticoid excess resulting specifically from a pituitary adenoma. Physical and biochemical abnormalities result from a state of prolonged hypercortisolism. Metabolic abnormalities associated with hypercortisolism include; glucose intolerance, hyperglycemia, hypokalemia, and hypertension. Hypovolemia and hypotension may be seen in cortisol deficiency states (such as Addison’s disease or pituitary dysfunction); but not, however, in a patient diagnosed with Cushing’s disease.

65
Q
Which drug should be avoided in the patient with pre-existing renal dysfunction?
A. Ketorolac (Toradol)
B. Midazolam (Versed)
C. Methohexital (Brevital)
D. Droperidol (Inapsine)
A

Answer: A
Rationale:
Barbiturates and benzodiazepines both reduce glomerular filtration rate but can be used in patients with renal failure. Doses need to be reduced and the anesthetic medication titrated to effect. Opioids, such as morphine and meperidine have active metabolites that are renally excreted and require cautious administration. The glucoronidated morphine compound is more potent than morphine itself. Droperidol, although, not generally used in outpatient anesthesia currently produces an -adrenergic blocking effect that preserves renal blood flow and hemodynamics. Ketorolac or other NSAIDs are prostaglandin inhibitors that interfere with the prostaglandin mediated renal vasodilatation and has been shown to produce acute renal failure.
Reference:
Altee John L. Complications in Anesthesia. WB Saunders 1999
Roizen MF, Fleisher LA. Essence of Anesthesia Practice. WB Saunders 1997

66
Q
A 65 year-old 45 kg frail cardiac transplant patient presents for extractions. Local anesthesia with mepivicaine 3% 270 mg and sedation with midazolam 1.25 mg i.v. with a 50% mixture of nitrous oxide and oxygen is used. The patient becomes hypotensive and bradycardic with a narrow complex QRS complex. What drug or intervention would you use in the management of this patient?
A. Flumazenil 4 mg IV push
B. Fluid bolus of 500 mg normal saline
C. Dopamine 200 μg/min IV infusion
D. Atropine 0.5mg IV push
A

Answer: C
Rationale:
The cardiac transplant patient’s chronic problems may include diffuse accelerated atherosclerosis leading to generally non-anginal myocardial ischemia and decreased cardiac output. Additionally, the chronotropic response is markedly decreased in the denervated transplanted heart. This patient is suffering from symptomatic bradycardia. One contributing factor could be the local anesthetic. Although the dosage of mepivicaine administered is within the upper acceptable recommended guidelines the practitioner must take into consideration the patient. Cardiovascularly compromised patients may be on several drugs that interact with intraoperatively administered medications. It would not be uncommon for a patient with this history to be taking an additional local anesthetic type dysrhythmic drug acting on her myocardium (such as procainamide.) Additionally, a local anesthetic injection into or near the pterygoid plexus can cause a precipitous rise in local anesthetic blood levels, especially with a lack of vasoconstrictor. The appropriate first step would be external pacing if available. If this is not available, a dopamine infusion titrated between 5-20 μg/kg/min would give both alpha and beta effects to raise heart rate and blood pressure. Since this patient’s transplanted sinus node is denervated, atropine would not reverse her bradycardia. Flumazenil should be given in incremental doses of 0.05-0.1 mg slowly. For a frail patient, a 500 ml fluid bolus might significantly increase cardiac preload and further decrease cardiac output.
Reference:
ACLS Provider manual, 2001 American Heart Association, pp145-155
Weinberg, GL: Basic Science Review of Anesthesiology McGraw-Hill, New York, 1997 p 25

67
Q

A 40 year-old male Type II diabetic arrives at your office for the extraction of multiple teeth under intravenous sedation. He is combative and disoriented. He has been NPO for six hours. He took his diabetic medications (acarbose and glyburide) this morning. A finger stick glucose is 50. What is the next appropriate step?
A. Fruit juice PO
B. Intramuscular 50% Dextrose
C. Intramuscular glucagon
D. Dextrose tablet dissolved sublingually

A

Answer: C
Rationale:
Hypoglycemia after fasting while on oral hypoglycemics is a concern in office anesthesia settings, and patients must understand that they should avoid immediate preoperative hypoglycemic medications in that setting. This patient’s medications include a second generation sulfonylurea (glyburide) which increases endogenous insulin release; and an α- glucosidase inhibitor (acarbose) which blocks intestinal breakdown of fructose and dextrose into absorbable glucose. In this case intake of fructose-containing juice is not indicated. Although one could consider oral administration of glucose containing tablets or solution, oral administration in a disoriented patient is also relatively contraindicated. Intramuscular administration of the very hyperosmotic 50% dextrose would cause extreme muscle irritation; its use should be limited to intravenous administration in a large vein. Dextrose is not absorbed significantly through sublingual epithelium. Glucagon 1 mg IM would be indicated. This would be absorbed and cause hepatic glycogenolysis and gluconeogenesis, raising blood glucose levels in about 15 minutes. Glucagon may cause significant nausea.
Reference:
Sarasin D: Ambulatory anesthetic management of the patient with diabetes. Oral Maxilofac Surg Clin N Amer 11(4): 589-99, 1999
Washington Manual of Medical Therapeutics, 30th ed, Liippincott, Philadelphia, 2001 pp463-66

68
Q

A 72 year-old male presents for removal of tooth #3. His written medical history is significant for stable angina, congestive heart failure, type 2 diabetes mellitus, and coronary stent placed after myocardial infarction 3 months ago. He is taking metoprolol and furosemide. A history and physical examination reveals the following: no chest pain with mild exercise, reduced carotid pulses, audible 3rd and 4th heart sound, and inspiratory basilar rales. The most influential determinant of this patient’s perioperative cardiac risk status is:
A. stableangina.
B. coronary stent placement following MI 3 months ago.
C. congestive heart failure.
D. diabetes mellitus.

A

Answer: C
Rationale:
This patient has signs of acute congestive heart failure. Physical signs of left ventricular CHF include: reduced carotid pulsations, diffuse and laterally displaced apical impulse, palpable and audible 3rd and 4th heart sounds, accentuated pulmonic 2nd sound, inspiratory basilar rales, and right-sided pleural effusion. There are several factors which contribute to determining a patient’s overall preoperative cardiac risk. As shown in the ACC/AHA 2002 Guidelines Update on Perioperative Cardiovascular Evaluation for Noncardiac Surgery and Goldman et al. (Multifactorial index of cardiac risk in non-cardiac surgical procedures. N Engl J med 1977;297:845-50.), the preoperative factor which had the highest predictive value for an adverse cardiac event is the patient’s acute congestive heart failure.
Reference:
ACC/AHA 2002 Guidelines Update on Perioperative Cardiovascular Evaluation for Noncardiac Surgery . Circulation. March 5,2002
Goldman et al.: Multifactorial index of cardiac risk in non-cardiac surgical procedures. N Engl J med 1977;297:845-50.

69
Q

In a patient with interstitial lung disease (ILD) why would one expect the FEV1/FVC ratio to be normal?
A. Vital capacity is reduced in ILD
B. FEV1 is normal in a restrictive pattern
C. ILD does not alter PFT’s
D. ILD affects only the diffusing capacity of CO2

A

Answer: A
Rationale:
In ILD a known or unknown agent stimulates macrophage induced alveolitis. This results in a loss of functioning alveolar capillary units and an accumulation of fibroblasts. The accumulation of collagen in the alveolar interstitium produces a thickened, scarred and eventually small lung.
The FEV/FVC ratio is typically normal in ILD due to a tendency for patients to have small lung volumes. Small lung volumes (VC – vital capacity, TLC – total lung capacity) will be noted on both PFT examination and Chest radiographs. While FEV1 is decreased in ILD, the FVC is decreased at the same time resulting in a normal FEV/FVC ratio.
Reference:
Braunwald, E, et al: Interstitial Lung Disease: Harrison’s Principles of Internal Medicine. McGraw-Hill Co.
Alex, CG and Tobin, MJ: Assessment of Pulmonary Function in Critically Ill Patients, in Textbook of Critical Care, Shoemaker, WC. 4th Edition. W B Saunders Co. 2000.

70
Q
40. Four days after discharge after a bimaxillary osteotomy, a 16 year-old female returns to your office complaining of malaise. You note considerable pitting edema of the lower extremities and rales on chest auscultation. Review of the anesthesia record shows approximately 1000 ml of blood loss and induced hypotension to a mean arterial pressure of 60 during the maxillary downfracture. Surgical time was 7 hours. No blood products were administered. Which of the following would you expect to find?
A. Serum osmolality 250 mOsm/L
B. Serum creatinine 0.9 mg/dL
C. Intense diruesis
D. Hypokalemia
A

Answer: A
Rationale:
This patient’s clinical presentation is consistent with acute intrinsic renal failure due to intraoperative hypoperfusion. Renal ischemia brought about by loss of red cell mass and hypotension in this prolonged procedure caused glomerular and tubular damage. The classic initial presentation of acute renal failure is oliguria and fluid retention, causing a decreased serum osmolality (normal = about 275mOsm/L.) Retention of nitrogenous waste products by lack of renal excretion would cause increases in serum creatinine (normal = 0.5-1.2 mg//dL.) In the early phase of ARF, typically an oliguric phase occurs for the first 1 to 2 weeks. If there is a renal recovery, intense dieresis usually follows this. Early ARF also generally shows metabolic acidosis and subsequent hyperkalemia.
Treatment of ARF includes intense fluid and electrolyte management, avoiding nephrotoxic medications, dialysis if necessary, and prevention of infection (which is more common during the diuretic phase of ARF.)
Reference:
Washington Manual of Medical Therapeutics, 30th ed, Liippincott, Philadelphia, 2001 pp258-263
Fauci et al (eds.) Harrison’s Principles of Internal Medicine 14th Ed, McGraw-Hill, New York, 1998 pp1511-1513

71
Q

A patient develops STATUS asthmaticus after an anesthetic and requires emergent endotracheal intubation and mechanical ventilation for respiratory failure. Once transferred to the ICU, it is noted that your patient has a depressed cardiac output, hypotension and hypercapnia. What is the likely explanation for this constellation of findings?
A. Secondary bacterial pneumonia
B. Dynamic hyperinflation by intrinsic positive end-expiratory pressure (air trapping)
C. Beta agonist toxicity
D. Inadequate fluid resuscitation

A

Answer: B
Rationale:
This constellation of findings is seen with intrinsic/auto PEEP due to dynamic airway collapse (“air trapping”). The FRC is increased by recruiting more alveoli and by increasing alveolar size. Intrinsic (auto) PEEP occurs when the inspiratory/expiratory time ratio approaches 1:1. Hyperinflation of the thoracic cavity results in increased intra- thoracic pressure and subsequently a decrease in cardiac output. Ideally, the I/E ventilator ratio should be maintained at 1:3 or 1:4 for obstructive pulmonary patients.
Reference:
Ninane V, et al: Intrinsic PEEP in patients with chronic obstructive lung disease: Role of expiratory muscles. Am Rev Respir Dis 148:1037, 1993.
Alex, CG and Tobin, MJ: Assessment of Pulmonary Function in Critically Ill Patients, in Textbook of Critical Care, Shoemaker, WC. 4th Edition. W B Saunders Co. 2000.

72
Q

A 68 year-old male with stable atherosclerotic heart disease develops significant hypotension, BP 75/35 with a heart rate of 110 following induction of general anesthesia. What would be the most appropriate first line pharmacotherapy?
A. Atropine
B. Phenylephrine C. Ephedrine
D. Epinephrine

A

Answer: B
Rationale:
Phenylephrine is an alpha agonist. It will increase SVR and may cause a reflex bradycardia, which would be beneficial for cardiac function.
Ephedrine and epinephrine would both increase heart rate due to their beta stimulation and given this patients medical history that would not be desirable.
Atropine is an anticholinergic drug, would increase heart rate and compromise the ischemic heart.
Reference:
Firestone, Lebowitz and Cook: Clinical Anesthesia Procedures of the Massachusetts General Hospital , Third edition, P. 634-5
Barash, Cullen and Stoelting, Handbook of Clinical Anesthesia, Fourth edition, P.871

73
Q
The patient is a 72-year old male who lives alone and has recently started to exhibit signs of memory loss. He is otherwise healthy except for multiple medications including: levothyroxine (thyroid replacement), vytorin (hypercholesterelemia) and hydrochlorothiazide (diuretic). He is cleared for surgery and anesthesia for multiple extractions and mandibular tori removal. Intraoperatively, he develops a significant tachycardia and severe hypotension. A fluid challenge and reduction in anesthetic agents do not alter these conditions. The most likely diagnosis is:
A. fluidoverload.
B. malignant hyperthermia.
C. thyrotoxicosis factitia.
D. pheochromocytoma.
A

Answer: C
Rationale:
Thyroid storm is an abrupt exacerbation of hyperthyroidism caused by sudden release of thyroid hormone (T4) into circulation by accident or an otherwise clinical presentation. Clinical symptoms mimic malignant hyperthermia. There is tachycardia, dehydration, hyperthermia and shock. This would be high on the list of possible causes due to the sudden loss of memory, the use of multiple medications and the solitary situation.
Reference:
Stoelting, R.K. and Dierdorf, S.F: Anesthesia and Co-Existing Disease. 4th Edition, 2002. Philadelphia, Churchill Livingstone. Pp. 415-7
Beers, M. and Berkow, R.: The Merck Manual. 17th Edition, 1999. Whitehouse Station, Merck Research Laboratories. P. 89.
Cole, D.J. and Schlunt, M.: Adult Perioperative Anesthesia 2004. Philadelphia, Mosby pp.71-73

74
Q
In children, blood pressure is determined almost entirely by which of one of the following physiologic parameters?
A. StokeVolume
B. Myocardial Contractility
C. Systemic Vascular Resistance
D. Heart Rate
A

Answer: D
Rationale:
A child’s required cardiac output can be double that of an adult; it is related to an increased metabolic rate and increased oxygen consumption. Blood pressure is dependent on stroke volume, myocardial contractility, systemic vascular resistance and heart rate. In the child, stroke volume, myocardial contractility, or systemic vascular resistance are relatively invariable. Blood pressure is almost entirely rate dependent. Since children show a tendency toward vagal response to many stimuli; monitoring of the heart rate is critical. Bradycardia in children invariably leads to hypotension.
Reference:
Dembo, Jeffrey B., Pediatric Anesthesia pg.837 in Oral & Maxillofacial Surgery Clinics of North America. W.B. Saunders Co., Nov. 1992.
Kelly, John P., Dembo, Jeffrey,B ., in Anesthesia Section, OMS Know ledge Update, Vol. 1 Part 1, August 1994, Pg. 42

75
Q

While under anesthesia for an iliac crest bone graft a patient is noted to become suddenly tachycardic, hypotensive and hypoxic. The capnography reveals decreasing end-tidal CO2.The procedure is terminated. A 12-lead EKG is obtained that reveals new findings of right-axis deviation, peaked T-waves and an incomplete right bundle-branch block. The most likely diagnosis for this situation is:
A. right to left cardiac shunting.
B. malignant hyperthermia.
C. pulmonary embolism.
D. pulmonary edema secondary to left ventricular failure.

A

Answer: C
Rationale:
Massive pulmonary embolism/ cor pulmonale are associated with right ventricular failure, arterial hypoxemia and decreasing end-tidal CO2. The V/Q mismatch results in areas of wasted pulmonary ventilation. Conversely, a right to left shunt is an extreme V/Q abnormality when there is perfusion and no ventilation at all. A decrease in CO2 on intra- operative capnography suggests decreased lung perfusion. The EKG will rule out acute AMI and show right heart strain.
Reference:
Dehring, DJ, Arens JF: Pulmonary Thromboembolism: Disease recognition and patient management. Anesthesiology 73:146, 1990
Divekan, VM et al. Pulmonary embolism during anesthesia: Case Report. Can Anesthes Soc J 28:277, 1981

76
Q
Which of the following signs and/or symptoms are associated with venous thrombosis?
A. Homans sign
B. Levine’s sign
C. Quinke’s sign
D. Psoas sign
A

ANSWER: A
RATIONALE:
Homan’s sign: calf pain with forcible dorsiflexion of the foot, associated with lower extremity deep venous thrombosis. Levine’s sign: clenched fist over the chest while describing chest pain: associated with angina and acute myocardial infarction. Quinke’s sign: alternating blushing and blanching of the fingernail following light compression: seen in aortic regurgitation. Psoas sign (iliopsoas test): extension and elevation of the right leg produces pain in cases of inflammation of the psoas muscle: indicative of appendicitis.

77
Q

In taking the blood pressure on an extremely obese patient, the standard size cuff would result in a blood pressure reading that is:
A. accurate
B. higher than the actual blood pressure
C. lower than the actual blood pressure
D. unreliable, since it is not possible to obtain an accurate blood pressure on an extremely
obese patient.

A

ANSWER: B
RATIONALE:
When considering the correct size of cuff, two pertinent points should be recalled: 1.) The inflatable bladder in the cuff should be able to completely encircle the arm with minimal overlap. 2.) The width of the bladder in the cuff should be approximately 20% greater than the diameter of the extremity used for the blood pressure cuff. Applying a cuff that is too small for an obese arm will produce a falsely elevated blood pressure reading; while applying too large a cuff on a thin arm will cause a falsely decreased blood pressure reading. Additionally, applying the cuff too loosely will produce a falsely elevated reading.

78
Q

Which of the following statements concerning cardiac output and myocardial work is true?
A. Preload represents passive ventricular wall stress and is best measured during systole
B. The primary determinants of afterload are the total peripheral resistance the heart muscle
must pump against and changes in intrathoracic pressure
C. Increasing heart rate is an efficient means of increasing myocardial work
D. Contractility is a direct measurement of the ability of the heart muscle to withstand
passive stretching

A

ANSWER: B
RATIONALE:
Cardiac afterload is indirectly measured through blood pressure and mean arterial
pressure. Increasing afterload (for example, via increasing peripheral vascular resistance or
intrathoracic pressure) or increasing heart rate increases myocardial oxygen consumption and work.
While preload does indeed represent passive ventricular wall stress, it is measured during diastole when the heart muscle wall is in its passive state. Preload is generally a reflection of the volume status of the patient. Increased heart rate is an inefficient means to increase cardiac output. Elevated heart rate is also potentially harmful in that it decreases the time that oxygen and nutrients can be delivered to the myocardial cells (diastolic perfusion time). Contractility is defined as the ability of the heart muscle to shorten with appropriate stimulation. With increased shortening of the muscle fibers during myocardial contraction, the heart can generate additional cardiac output more efficiently (an inotropic response) than by increases in heart rate (a chronotropic response).

79
Q

Which of the following concerning AV node conduction is true?
A. Modulation is achieved through nicotinic and cholinergic mechanisms.
B. AV conduction on the ECG is represented by the Q-T interval.
C. Digoxin enhances conduction speed.
D. No intrinsic automaticity is present at this node.

A

ANSWER: A
RATIONALE:
The vagus nerve provides cholinergic stimulation to the heart at the AV node and mediates a negative chronotropic effect. Catecholamines have the opposite effect and increase speed of impulse conduction through the AV node via nicotinic receptors. Catecholamines also cause an increase in myocardial inotropy.
In ECG tracings, the P-R interval represents the usual delay (0.20 secs) in conduction through the AV node. While digoxin is a positive ionotrope, it is also a negative chronotrope, decreasing the conduction velocity through the AV node. Although the automaticity of the AV node is usually masked by the more rapid impulses generated by the sino-atrial node, in the absence of atrial impulses the AV nodal junction often will generate depolarization at a rate of 40 to 60 impulses per minute.

80
Q

Which of the following concerning Wolff-Parkinson-White Syndrome is true?
A. Sigma waves may alter the P-R interval on ECG.
B. Conduction is via the bundles of His.
C. Rapid ventricular response may be controlled with digitalis.
D. Procainamide may decrease conduction through accessory pathways.

A

ANSWER: D
RATIONALE:
WPW is a syndrome of rapid ventricular response to atrial stimulation by conduction through the accessory Bundle of Kent, bypassing the AV node and therefore the ability of the AV node to control over-rapid atrial impulse conduction to the ventricles. A gradual upslope of the P-R interval, the delta wave, is an ECG characteristic of this disorder. Emergent control of atrio- ventricular tachycardic conduction is by synchronized cardioversion if the patient is unstable. Medical management includes those drugs that can decrease impulse transmission through the accessory pathway (procainamide, amiodarone.) Digitalis and verapamil increase AV node refractoriness to conduction and can increase conduction through the aberrant pathway, which can cause serious deterioration in cases of tachycardia of supraventricular origin. Definitive treatment of the stable patient includes radiofrequency ablation of aberrant pathways.

81
Q

Non-pathological heart sounds, S1 and S2 can be characterized by:
A. left heart valve closure usually louder than right.
B. splitting of S1 during inspiration.
C. fixed splitting of S2 in the adolescent.
D. splitting of S2 on expiration.

A

ANSWER: A
RATIONALE:
Left (mitral) valve closure is louder due to the higher pressure in the aorta and left heart. S1 splitting, which represents a significant difference in the timing of closure of the mitral and tricuspid valves, is usually due to a pathologic process such as pulmonic stenosis or right bundle branch block. Non-pathologic split of S2 can occur on inspiration. Paradoxical S2 split occurs on expiration, with the most common associated pathology being left bundle branch block. Fixed S2 split can be indicative of atrial septal defect or right ventricular failure.

82
Q

Ventricular dilation in congestive heart failure is the result of:
A. increased cardiac output.
B. increase in circulating catecholamines.
C. decrease in ventricular afterload.
D. increase in end-diastolic ventricular volume.

A

ANSWER: D
RATIONALE:
Congestive heart failure is defined as the inability of the heart to maintain a cardiac output that meets the demands of peripheral organs. Catecholamine output is initially increased to attempt to increase heart rate and contractive force in order to maintain cardiac output. However, this is also accompanied by an increase in peripheral vascular resistance causing increased afterload. Eventually the myocardium cannot compensate and the end-diastolic ventricular volume is increased, due to decreased cardiac output and increased end-diastolic volume blood in left ventricle prior to systole. Myocardial failure can be secondary to coronary artery disease, non- ischemic cardiomyopathy, or longstanding valvular problems such as aortic incompetence.

83
Q
Peripheral pedal edema and jugular venous distension are primarily characteristics of:
A. left heart failure
B. right heart failure
C. pulmonary edema
D. nephrotic syndrome
A

ANSWER: B
RATIONALE:
Right heart failure causes systemic venous congestion, resulting in jugular venous distension and causing peripheral edema from lymphatic stasis.
Left sided heart failure causes pulmonary vascular congestion, leading to pulmonary edema, dyspnea, orthopnea, and changes of pulmonary vasculature on chest radiographs. Nephrotic syndrome is a glomerulonephropathy causing severe proteinuria precipitating a large decrease in intravascular osmotic pressure and fluid loss to the interstitial tissue. While peripheral edema is a prominent symptom, intravascular volume depletion occurs and jugular venous distension is therefore not observed.

84
Q

Increased risk factors associated with patients diagnosed with congestive heart failure and managed with digitalis include all of the following except:
A. ejection fraction 20 mm Hg.
C. hyperkalemia
D. calcium channel blokers

A

ANSWER: C
RATIONALE:
Digitalis toxicity can be enhanced in the hypokalemic state and precipitate serious cardiac dysrhythmias.
Normal cardiac ejection fractions are 60-80%, and when

85
Q

When pre-operatively evaluating a patient that has an implanted permanent pacemaker, all of the following are true except:
A. Epicardial pacemakers do not require antibiotic prophylaxis for bacteremia-producing procedures.
B. A demand type pacemaker should be switched to a fixed rate mode to avoid interference of the pace making activity from intraoperative radiofrequency emitting equipment (example; electrocautery).
C. Dual chamber pacemakers can develop pacemaker-mediated tachycardia.
D. Patients with a pacemaker can not be defibrillated.

A

ANSWER: D
RATIONALE:
All current pacemakers allow defibrillation; however they should be checked for proper function after defibrillation. Demand pacemakers can undergo interference from any strong radiofrequency source, especially if it is grounded to the patient (such as electrocautery;) so demand pacemakers should be set on a fixed rate to avoid inappropriate interference with the demand function. Pacemaker-mediated tachycardia is a possible complication of dual- chamber(atrial and ventricular) pacing when the atrial lead senses retrograde depolarizations because of ventriculoatrial conduction. The resulting tachycardia often has a rate equal to the upper rate limit of the pacemaker. Pacemaker-mediated tachycardia can be eliminated by various reprogramming maneuvers, such as lengthening the post-ventriculoatrial refractory period.

86
Q

Which of the following statements regarding aortic stenosis is incorrect?
A. Aortic stenosis is typified by a midsystolic ejection murmur and a narrowed pulse pressure.
B. The triad of angina, syncope and congestive heart failure represents progression of symptoms associated with aortic stenosis.
C. The development of supraventricular arrhythmias including atrial fibrillation creates hemodynamic problems for the patient with aortic stenosis.
D. Hypotensive anesthesia for the aortic stenosis patient is cardioprotective by decreasing afterload and myocardial work.

A

ANSWER: D
RATIONALE:
Hypotension (reduced systemic vascular resistance ) does little to relieve the fixed afterload arising from a stenotic aortic valve; however hypotension lowers the diastolic coronary profusion gradient leading to myocardial ischemia. Therefore, induced hypotensive states are contraindicated in the patient with a stenoticaortic valve.
Aortic stenosis is characterized by a crescendo-decrescendo systolic murmur (which may radiate to the carotids) and narrowed pulse pressure. With left ventricular hypertrophy an apical thrust may be seen. The triad of angina, syncope and congestive heart failure correlate directly with mortality; the 50% survival data for these symptoms are 5,3, and 2 years respectively from the onset of symptoms without surgical treatment. Patients with aortic stenosis need the left ventricular filling obtained through a well timed atrial contraction. Supraventricular arrhythmias decrease ventricular filling (especially in the less compliant myocardium of left ventricular hypertrophy) and therefore decrease the amount of blood available for ejection past the stenotic aortic valve.

87
Q
Which of the following produces a diastolic murmur?
A. Aortic stenosis
B. Mitral regurgitation
C. Mitral valve prolapse
D. Mitral stenosis
A

ANSWER: D
RATIONALE:
Mitral stenosis produces a diastolic rumbling murmur. By auscultation one can hear an opening snap followed by a low-pitched diastolic rumble best heard at the apex. Diagnosis is confirmed by Doppler echocardiography. The most common cause of mitral stenosis is rheumatic fever. The first symptom of mitral stenosis is usually dyspnea on exertion as a result of pulmonary venous congestion secondary to elevated left atrial pressure.
The most common causes of aortic stenosis are rheumatic fever and congenital anomaly. Associated symptoms include syncope, dyspnea on exertion and angina. In the adult the physical findings are consistent with a systolic ejection (crescendo-decresendo) murmur and delayed pulse up-stroke. Diagnosis is confirmed with cardiac catheterization. Mitral regurgitation often is detected by a holosystolic rumbling murmur, while mitral valve prolapse yields a systolic click murmur.

88
Q

Which of the following statements regarding premature ventricular contractions PVC’s are true?
A. Unifocal PVC’s in patients without a previous cardiac history may indicate early signs of myocardial infarction
B. Six or more PVC’s in a minute, especially if they are multifocal are considered ventricular tachycardia.
C. They should always be treated promptly to avoid the risk of ventricular tachycardia or fibrillation.
D. They rarely occur in a normal, healthy individual.

A

ANSWER: B
RATIONALE:
Six or more PVC’s per minute are by definition ventricular tachycardia. Depending upon the clinical situation, antiarrhythmic therapy may be justified, especially if these are multifocal. Unifocal PVC’s in an otherwise healthy individual warrant investigation for nonspecific cardiac challenges such as hypoxemia, hypercarbia, acidemia, sympathetic surge, drug effects and electrolyte disturbances. They are, however, not indicative of impending myocardial infarction. Therefore an intelligent consideration of the clinical situation and a search for possible causes in the otherwise healthy patient should be performed rather than a “knee jerk” response of antiarrhythmic therapy.

89
Q
What is the maintenance fluid requirement of a healthy 70 kg adult who is restricted from oral intake NPO while awaiting surgery?
A. 60 cc/hr
B. 80 cc/hr
C. 110 cc/hr
D. 140 cc/hr
A

ANSWER: C
RATIONALE:
The calculation for fluid replacement for a healthy individual is as follows:
HOURLY CALCULATION
40 ml/hr for the first 10 kg of body weight
20 ml/hr for the 2nd 10 kg of body weight 10 ml/hr for each additional 10 kg
Total = 110 cc/hr
DAILY CALCULATION
1st 10kgx100ml=1000ml
2nd 10kgx50ml=500ml 50 kg x 20 ml = 1000 ml
Total = 2500 ml/24 hr = 104 ml/hr

90
Q
Initiators of hepatic cirrhosis include all of the following except:
A. Chronic cholestasis
B. Halothane
C. Uncontrolled diabetes mellitus
D. Right heart failure
A

ANSWER: C
RATIONALE:
Chronic biliary obstruction can cause cirrhotic liver changes. Halothane, by an immune- mediated reaction to metabolic byproducts, can cause a fulminant acute hepatic necrosis that may lead to cirrhosis in susceptible individuals. Prolonged severe right heart failure can lead to hepatic fibrosis and “cardiac cirrhosis.” Although the microangiopathy of uncontrolled diabetes mellitus can affect many organ systems, hepatic involvement is unusual.

91
Q

A patient with a history of renal impairment and a measured glomerular filtration rate of 20 ml/min could be expected to manifest with which of the following?
A. Microcytic hypochromic anemia
B. Compensatory respiratory hypoventilation
C. Low anion gap
D. Hypertension tendency

A

ANSWER: D
RATIONALE:
Glomerular filtration of 20 ml/min would be considered to have moderate to severe renal failure. Moderate to severe renal failure affects the rennin-angiotensin system causing hypertension. Lack of renally-produced erythropoietin in renal failure yields a normochromic, normocytic anemia by decreased red blood cell production. Renal failure also causes a high anion-gap metabolic acidosis, which often is accompanied by a compensatory respiratory hyperventilation.

92
Q
Which medication should be avoided in the thyrotoxic patient?
A. atropine
B. methimazole
C. potassium iodide 
D. propranolol
A

ANSWER: A
RATIONALE:
Thyrotoxicosis is manifested by a hyperadrenergic state including hypertension and tachycardia. Atropine would aggravate the cardiovascular effects of this disorder and should be avoided. Antithyroid medications such as methimazole and propothiouricil decrease thyroid hormone synthesis and decrease peripheral conversion of T4 to the more metabolically active T3. Initial intravenous potassium iodide actually decreases the acute release of T3 and T4 from the thyroid, although long-term it can increase iodine storage in the gland. Propranolol mitigates the cardiovascular effects of hyperthyroid activity and is used in acute management of the disease.

93
Q
All of the following conditions are seen in patients with severe untreated hypothyroidism except:
A. Dementia
B. Cardiac failure
C. Hypolipidemia
D. Coma
A

ANSWER: C
RATIONALE:
Untreated severe hypothyroidism manifests with altered mental status up to and including coma, heart failure, muscular weakness/lethargy, and hyperlipidemia especially low density lipoproteins often with advanced athlerosclerosis.

94
Q

All of the following may be observed in the patient with untreated adrenal insufficiency except:
A. decreased systemic vascular resistance
B. peaked T waves on ECG
C. hypernatremia
D. rales

A

ANSWER: C
RATIONALE:
Adrenal insufficiency can include both cortisol and aldosterone production. Lack of cortisol can lead to decreased systemic vascular resistance and hypotension, especially under physiologic stressors. In the face of this challenge in a cardiac-debilitated patient, high-output congestive heart failure can lead to rales being auscultated. Physiologically, aldosterone release is under control of the rennin-angiotensin system; and aldosterone promotes renal sodium and water retention and potassium excretion. Conversely, hypoaldosteronism can lead to hyponatremia due to sodium losses, and to hyperkalemia, which is manifested by peaked T waves on ECG.

95
Q
Which of the following is the initial treatment for diabetic ketoacidosis?
A. Insulin
B. Isotonic saline
C. Potassium chloride
D. Sodium bicarbonate
A

ANSWER: B
RATIONALE:
Restoration of fluids and electrolytes is the first resuscitative priority due to dehydration and sodium depletion. Initial hydration rapidly corrects plasma volume and increases the efficacy of later insulin therapy. Intracellular potassium depletion may be masked by near normal or slightly elevated serum potassium levels, especially in a volume-depleted patient. Therefore, following initial rehydration, insulin administration is also accompanied by titrated intravenous potassium with careful electrolyte measurements. Volume expansion and insulin administration usually resolves the metabolic acidosis from ketoacid production, and bicarbonate administration is not indicated in most cases.

96
Q
A normal glycosylated hemoglobin (Hemoglobin A1c) level is:
A. 4-6 %
B. 10-12 %
C. 15-18%
D. 20-25%
A

ANSWER: A
RATIONALE:
The major form of glycohemoglobin, termed hemoglobin A1c normally comprises only 4-6% of total hemoglobin. It would be higher in chronically hyperglycemic patients due to
condensation of glucose with free amino acids on the globin component of hemoglobin. Therefore, 2, 3, and 4 are too high for a healthy individual.

97
Q
Which of the following would be considered the drug of choice for treatment of severe pseudomembranous colitis?
A. Vancomycin
B. Cefazolin
C. Clindamycin
D. Metronidazole
A

ANSWER: D
RATIONALE:
Metronidazole is an antibiotic which is effective against Clostridium difficile which causes pseudomembranous colitis. Vancomycin, due to its cost and concerns of promoting vancomycin microbiologic resistance (especially by Staphylococcus strains) has limited its oral use to very severe, metronidazole-resistant C. difficile enterocolitis infections. Cefazolin and clindamycin disturb the balance of intestinal flora and have been implicated as causative agents in the development of this infection.

98
Q
What endocrine abnormality is often an associated sequela of chronic renal failure?
A. Secondary hyperparathyroidism
B. Primary adrenal insufficiency
C. Hypothyroidism
D. Primary hyperaldosteronism
A

ANSWER: A
RATIONALE:
With renal failure there is decreased glomerular filtration which results in an increased level of serum phosphate. This tends to cause serum calcium to be deposited in bone leading to a decrease serum calcium level. In response to low serum calcium the parathyroid glands are stimulated to secrete parathormone (PTH) which results in secondary hyperparathyroidism. Primary adrenal insufficiency usually is a result of an autoimmune disorder but may also result from cancer, infection, or trauma. Hypothyroidism may result from any failure along the pituitary-thyroid axis (hypothalamus failure to release thyroid releasing hormone, adenohypophyseal failure to release thyroid stimulating hormone, or thyroid secretory failure.) Primary hyperaldosteronism results from adrenal cortical hyperplasia (specifically of the zona glomerulosa) or an aldosterone- secreting adenoma of the adrenal gland. None of the latter three states is a sequela of chronic renal failure.

99
Q
Increased anion gap may be found in :
A. Hyperkalemia
B. Multiple myeloma
C. Hypoalbuminemia
D. Ketoacidosis
A

ANSWER: D
RATIONALE:
Anion gap gives information concerning “unmeasured” serum anions. Diabetic ketoacidosis is the most common cause of an increased anion gap. Hyperkalemia, increased proteinaceous cation in multiple myeloma, and decreased proteinacious anion in hypoalbuminemia will all cause a decreased anion gap.

100
Q

Which of the following concerning the management of the Parkinson’s disease patient is false?
A. Sialorrhea and cardiac sphincter dysfunction increase the incidence of pulmonary aspiration.
B. Ephedrine should be avoided if the patient takes selegiline.
C. Levodopa should be discontinued 24 hours prior to neuromuscular blocking agents.
D. Levodopa therapy may lead to hypovolemia and dyskinesia.

A

ANSWER: C
RATIONALE:
Levodopa has a short half-life and withdrawal prior to surgery can precipitate muscle rigidity and attendant difficulties in respiration and handling the patient. Sialorrhea, esophageal and laryngeal dysfunction increase aspiration risk in Parkinson’s patients. Selegiline is a monoamine oxidase-B inhibitor that decreases dopamine catabolism, and mitigates Parkinsonism’s decreases in dopamine in the caudate nucleus and the putamen. Ephedrine can precipitate an adrenergic crisis in MAO-B treated patients. Levodopa increases the activity of the rennin-angiotensin system, potentiating hypovolemia.

101
Q

Which of the following statements regarding myasthenia gravis is incorrect?
A. Initial symptoms often include diplopia and ptosis.
B. The etiology involves decreased acetylcholine secretion at the neuromuscular junction.
C. Edrophonium is useful in diagnosis of this disorder.
D. Physiologic stress can exacerbate clinical symptoms, including respiratory muscle failure.

A

ANSWER: B
RATIONALE:
Myasthenia gravis is an autoimmune disease characterized by decreased acetylcholine receptors at the neuromuscular junction which leads to muscle weakness even with normal acetycholine secretion. . Ocular muscles are often the first affected, with diplopia and ptosis especially after repetitive eye activities. Stressors such as infection or surgery can exacerbate muscle weakness, including respiratory muscles. Edrophonium, an acetylcholinesterase inhibitor, often brings rapid relief from muscle weakness; however false negative and positive test results are not uncommon.

102
Q

Which of the following statements concerning myotonic dystrophy is true?
A. ECG abnormalities are uncommon.
B. Inheritance is autosomal recessive.
C. Initial cardiac involvement most often is hypokinesis.
D. Extremity weakness progresses from distal to proximal.

A

ANSWER: D
RATIONALE:
Distal-to-proximal weakness is the most common progression pattern, although myotonia and stiffness may occasionally predominate. The inheritance is autosomal dominant. Predominant cardiac involvement is by fatty degeneration and fibrosis of specialized cardiac conductive tissue (sino-atrial and atrio-ventricular nodes and His-Purkinje system.) This makes dysrhythmias the most common cardiac pathology accompanying myotonic dystrophy.

103
Q

Wernicke’s encephalopathy :
A. may be precipitated in susceptible individuals by saline infusion.
B. is due to folic acid deficiency.
C. is characterized by ophthalmoplegia and ataxia.
D. is often anteceded by mental status changes.

A

ANSWER: C
RATIONALE:
Often the initial presenting signs of Wernicke’s encephalopathy include abducens palsy, horizontal diplopia, nystagmus, and strabismus; and ataxia may cause ambulation difficulties. These neuromuscular signs often antecede mental changes. The disorder is caused by deficiency of thiamine (vitamin B1); administration of which can quickly reverse ocular symptoms but often has little effect on mental changes once they have occurred (such as anterograde and retrograde amnesia, apathy, drowsiness, confusion.) Glucose infusion into a thiamine-depleted individual (generally alcoholics) can precipitate Wernicke’s syndrome and all alcoholics receiving glucose infusions should have concomitant thiamine administration (50-100 mg immediately and then daily.)

104
Q

Which of the following concerning posttraumatic diabetes insipidus is true?
A. A hypertonic polyuria ensues.
B. Extreme dehydration may accompany hyponatremia.
C. An adenohypophyseal hormone analog is an effective treatment.
D. The targets of treatment are renal collecting ducts.

A

ANSWER: D
RATIONALE:
Traumatic diabetes insipidus results from a decreased secretion of antidiuretic hormone from the neurohypophysis (posterior pituitary.) The other neurohypophyseal hormone is oxytocin. Adenohypophyseal (anterior pituitary) hormones include follicle stimulating hormone, leuteinizing hormone, thyroid stimulating hormone, somatostatin, melanocyte stimulating hormone, and adrenocortical stimulating hormone.
Lack of ADH causes a decrease in water resorption from the renal collecting ducts, resulting in a voluminous hypotonic urine production, causing dehydration manifested by serum hyperosmolarity and hypernatremia. Treatment usually includes administration of intravenous or intranasal desmopressin, an analog of ADH that is almost devoid of vasopressor effects (unlike vasopressin.)

105
Q

Which of the following may be indicated in the treatment of acute intracranial injury?
A. Ventilation-induced serum pCO2

A

ANSWER: D
RATIONALE:
Osmotic diuresis with IV mannitol is commonly used to decrease intracranial pressure from acute head injury. There are no studies confirming the benefit of glucocorticoid use in reducing elevated intracranial pressure from acute head injury. Judicious and limited hyperventilation may be used to temporarily bring pCO2 down to 30 mm Hg but prolonged periods of pCO2

106
Q

The pathophysiology for Horner’s syndrome is:
A. interruption of preganglionic parasympathetic fibers.
B. Interruption of postganglionic sympathetic fibers.
C. aberrant conduction between motor branches of cranial nerve V and cranial nerve III.
D. traumatic or pathologic changes in the ciliary ganglion.

A

ANSWER: B
RATIONALE:
Horner’s syndrome is the result of disruption of sympathetic innervation to the orbital region characterized by miosis (unopposed parasympathetic-mediated papillary constriction;) upper lid ptosis (by loss of sympathetic innervation to Mueller’s muscle;) enophthalmos (either “apparent” by ptosis, or more rarely by atrophy of orbital contents if the syndrome occurs in a young patient or is longstanding;) and more inconsistently ipsilateral facial anhidrosis (lack of sweating.) Aberrant conduction (usually via a congential anomaly) between the motor branches cranial nerve V to the terminal branches of cranial nerve III produces Marcus Gunn syndrome (“jaw-winking” syndrome,) characterized by resting lid ptosis and upward motion of the superior lid with mandibular movement. Interruption of the parasympathetic fibers of the ciliary ganglion (where parasympathetic fibers to the orbit synapse) would result in pupillary dilation (mydriasis.)

107
Q
A patient with a left homonymous heminanopsia may indicate a lesion in the:
A. optic chiasm
B. right optic radiation
C. right optic nerve
D. left optic tract
A

ANSWER: B
RATIONALE:
Lesions of the optic radiation and of the optic tract produce a contralateral hemianopsia (loss of the contralateral field of vision in both eyes; in this case, a right optic radiation lesion causing a left homonymous hemianopsia.) Optic chiasm lesion produce a bitemporal hemianopsia (loss of temporal, i.e. lateral, field of vision in both eyes.) Lesions of the optic nerve produce an ipsilateral blind eye.

108
Q
At which parasympathetic ganglion do ocular preganglionic fibers synapse with postganglionic fibers?
A. Superior cervical
B. Pterygopalatine
C. Otic
D. Ciliary
A

ANSWER: D
RATIONALE:
Parasympathetic ganglia are present near the target organ, unlike sympathetic where the ganglia are near the spinal cord. Parasympathetics to the globe arise from cranial nerve III and synapse at the ciliary ganglion, just posterior to the globe. The otic ganglion synapses parasympathetic nervous system fibers from cranial nerve IX and supplies the parotid gland. The pterygopalatine ganglion receives presynaptic PSNS fibers from cranial nerve VII and supplies the secretory glands of the palate and nasal cavity. The superior cervical ganglion is sympathetic.

109
Q

In comparing the characteristics of rheumatoid arthritis (RA) and osteoarthritis (OA), which of the following is incorrect?
A. RA has a significant inflammatory component while OA does not.
B. RA usually presents with multiple symmetric joint involvement; OA usually involves
only one or two joints initially.
C. Both processes have associated systemic manifestations, including fatigue, weakness, and
malaise.
D. There are no laboratory tests that are pathognomonic or accurately diagnostic for either
process.

A

ANSWER: C
RATIONALE:
Signs and symptoms of RA include multiple symmetric joint involvement, a significant inflammatory component, morning joint stiffness lasting more than one hour, symmetric swelling of the proximal interphalangeal joints, systemic manifestations of fatigue, weakness, and malaise. In comparison, OA is characterized by involvement of only one or two joints or joint groups (at least initially) morning stiffness lasting less than 15 minutes, and the initial hand lesions usually involve the distal interphalangeal joints. OA has no systemic involvement. There are no laboratory tests that are specifically pathogonomic for either process, although there area a number of serum markers that may accompany RA (such as elevations of rheumatoid factor, erythrocyte sedimentation rate; and a normochromic, microcytic anemia.)

110
Q

In obstructive pulmonary disease, which of the following changes in the total lung capacity (TLC) and residual volume (RV) occurs?
A. TLC is normal or decreased; RV is decreased
B. TLC is normal or decreased; RV is increased
C. TLC is normal or increased; RV is decreased
D. TLC is normal or increased; RV is increased

A

ANSWER: D
RATIONALE:
In obstructive pulmonary disease, total lung capacity may be normal or increased and residual volume is increased; both by air entrapment and emphysematous changes. Vital capacity is often decreased as the amount of nonventilated or poorly ventilated lung volume increases.

111
Q
Which of the following medication is least indicated for acute control of asthma?
A. Cromolyn sodium
B. Metaproterenol
C. Dexamethasone
D. Diphenhydramine
A

ANSWER: A
RATIONALE:
Chromolyn sodium is a mast cell stabilizer preventing the release of histamine in the mucosa of the tracheobronchial tree when used chronically, but acts too slowly to be useful on an emergent basis. Metaproterenol is a beta agonist used to dilate the airway and reverse bronchoconstriction. Dexamethasone may be used intravenously to control the mucosal inflammatory component of acute asthma. Diphenhydramine, a histamine antagonist, may be administered concomitantly with a beta agonist and a steroid to decrease histamine-mediated bronchoconstriction and mucosal edema.

112
Q
Which of the following is a form of restrictive lung disease?
A. Asthma
B. Bronchiectasis
C. Cystic fibrosis
D. Sarcoidosis
A

ANSWER: D
RATIONALE:
Sarcoidosis is restrictive because sarcoid lesions cause a decreased compliance of lung parenchyma, restricting the amount of lung capacity. Asthma, bronchiectasis, and cystic fibrosis increase airway ventilatory resistance (especially during exhalation) and air entrapment, and are therefore obstructive diseases.

113
Q

Which of the following statements about the risk of bleeding in a patient who is to under go an extraction is false?
A. Cancer chemotherapeutics may cause thrombocytopenia.
B. Alcohol abuse may cause changes in prothrombin time and partial thromboplastin time.
C. Broad-spectrum antibiotic therapy may affect factors II, VII, IX, and X as well as
prothrombin time.
D. Partial thromboplastin time is the most appropriate test for acquired coagulopathies.

A

ANSWER: D
RATIONALE:
The best screening test for aquired coagulopathies is the prothrombin time (PT) or the International Normalized Ratio (INR). To have a reasonably good chance of achieving hemostasis by local measures, the patient’s PT should be within 1.5 times the control time and the INR below 2.0. Myelosuppression, as manifested by leukopenia, thrombocytopenia, and anemia, are common sequelae of cancer chemotherapy. Within 2 weeks of the beginning chemotherapy, the white blood cell count falls to an extremely low level. Thrombocytopenia can be marked, and spontaneous oral cavity bleeding may occur. Recovery from myelosuppression is usually complete 3 weeks from cessation of chemotherapy. Alcohol abuse may cause liver cirrhosis and thus decrease production of the liver dependent coagulation factors, thus obtaining a PT and PTT may be prudent in this patient population. Broad spectrum antibiotics may cause a change in the intestinal flora, which may decrease vitamin K production. Vitamin K is necessary for the liver to produce adequate quantities of coagulation factors II, VII, IX, and X. If the patient has a history of prolonged broad spectrum antibiotic therapy, the surgeon should be suspicious of decreased hemostasis.

114
Q

Which of the following statements concerning glucagon is correct?
A. Storage is in pancreatic beta islets
B. Clinical use is for acute hyperglycemia.
C. Primary target is myelocytes.
D. Secretion is under sympathetic control.

A

ANSWER: D
RATIONALE:
Pancreatic sympathetic innervation stimulates adrenergic mediated glucagon release. Glucagon is released from alpha pancreatic islets; insulin is released by beta pancreatic islets. Glucagon primarily targets hepatocytes to cause glycogenolysis and fatty gluconeogenesis. Glucagon secretion is in response to hypoglycemia and to increased demand for glucose (such as times of physiologic stress.)

115
Q

Bleeding time is an important evaluation of hemostasis in alcoholic chronic liver disease because of all of the following except:
A. hepatic platelet sequestration
B. qualitative thrombocyte dysfunction
C. decreased coagulation factor production
D. hypersplenism

A

ANSWER: C
RATIONALE:
Bleeding time is the best single screening test for qualitative or quantitative platelet disorders. Chronic alcoholic liver disease may be characterized by hepatic or hypersplenic platelet sequestration (quantitative); while cirrhotic changes in the liver decrease platelet function (qualitative). Although severe hepatic dysfunction can cause decreases in coagulation factor production, this is measured by other tests such as prothrombin time and activated partial thromboplastin time.

116
Q

Which of the following statements concerning vasopressor use in ventricular fibrillation is correct?
A. Escalating repeat epinephrine dosing may increase survival compared to non-escalating dosing.
B. Vasopressin stimulates peripheral α-1 receptors more effectively than does epinephrine.
C. Antidiuretic hormone lacks cardiac inotropic effects.
D. Vasopressin may be given by the endotracheal route.

A

ANSWER: C
RATIONALE:
Vasopressin, the endogenous antidiuretic hormone, has marked vasoconstrictive properties when given in doses much greater than found in vivo. Its vasoconstrictive activity is by binding to the V1 receptor, with no activity at adrenergic alpha or beta receptors. Since it has no beta-adrenergic affinity, it has no effect on cardiac inotropy or chronotropy. No consistent evidence shows that escalating epinephrine dosing increases survival-to-discharge odds from ventricular fibrillation, so that escalating dosing is considered acceptable but not recommended by the American Heart Association. Vasopressin is not recommended for endotracheal administration; common ACLS medications that can be given by this route are : lidocaine, atropine, and epinephrine. Naloxone can also be give via the endotracheal tube.

117
Q

When considering a general anesthetic for an 18 year old with Duschenne’s muscular dystrophy, which statement is true?
A. Restrictive lung disease from muscle atrophy-mediated kyphoscoliosis is often present.
B. Steroidal depolarizing muscle relaxants are contraindicated.
C. Volatile anesthetics are general anesthetic agents of choice.
D. There is a female preponderance

A

ANSWER: A
RATIONALE:
The progressive muscle atrophy seen in Duschenne’s muscular dystrophy causes vertebral and rib contractures, resulting in kyphoscoliosis and a restrictive ventilatory defect. Although depolarizing muscle relaxants are contra-indicated due to the possible induction of hyperkalemia, steroidal muscle relaxants are non-depolarizing. Duschenne’s muscular dystrophy patients have a relatively high incidence of malignant hyperthermia, which may be triggered by depolarizing muscle relaxants and volatile anesthetics; therefore both are contra- indicated in this patient group. Duschenne’s muscular dystrophy is an X-linked recessive disorder and therefore more common in males.

118
Q

When considering instituting total parenteral nutrition, which of the following statements is true?
A. Peripheral venous access is appropriate if a large bore vein is present.
B. Bolus administration can precipitate a hypo-osmotic state.
C. Intrinsic protein stores are exhausted prior to carbohydrate and fat.
D. Acute pancreatitis is an indication for this therapy.

A

ANSWER: D
RATIONALE:
Total parenteral nutrition is indicated for many conditions which may cause malabsorption of nutrients from the gastrointestinal tract such as acute pancreatitis. Central venous access is necessary, as peripheral access cannot accommodate the volumes of concentrated fluid necessary for total parenteral support of nutrition. Injudicious bolus administration of TPN fluid (which often has a concentration greater than 1000 mosm/L) can cause a hyperosmotic state. When calculating a particular TPN prescription, one must consider that intrinsic carbohydrate stores are exhausted in less than 24 hours, while
the body’s protein and fat stores may be available for weeks.

119
Q
Which of the following medications may trigger asthmatic symptoms?
A. Atropine
B. Ipatropium
C. V aldecoxib
D. Isoetharine
A

ANSWER: C
RATIONALE:
Valdecoxib (Bextra) is a cyclo-oxygenase-2 inhibitor. Any inhibitor of prostaglandin synthesis (such as nonsteroidal anti-inflammatory drugs) can cause an increase in leukotrienes which cause bronchoconstriction.
Atropine, being an anticholinergic, was formerly used to decrease bronchoconstriction in asthma but is no longer used because of its systemic side effects. Ipatropium bromide(Atrovent) is an inhaled anticholinergic used in chronic refractory asthma and in chronic obstructive pulmonary disease. Isoetharine (Bronkosol) is an inhaled B2 agonist used for bronchodilation as a nebulized solution.

120
Q

A 36-year-old obese female is in your office requesting a general anesthetic for extraction of a carious tooth. Your primary concern in regards to her obesity and pulmonary function is:
A. a decreased FEV1.
B. a decreased functional residual capacity.
C. a decreased minute ventilation.
D. a decreased residual volume.

A

Answer: B

Rationale:
Morbid obesity is characterized by reductions in functional residual capacity (FRC= volume remaining in the lungs after a normal quiet expiration), expiratory reserve volume (ERV=volume of air that can forcefully expired after a normal resting expiration) and total lung capacity (TLC). These changes have been attributed to mass loading and splinting of the diaphragm. Anesthesia compounds these problems and impairs the ability of the obese to tolerate periods of apnea. Residual volume consists of the gases remaining in the lung after a forced expiration and is less variable than other parameters. FEV1 is the forced expiratory volume in 1 second and is most often used as a determinant of inflammation and small airway obstruction in obstructive lung diseases such as asthma.

121
Q
Which of the following is the least likely cause of acute respiratory distress in the traumatized patient who is conscious when presenting to the emergency department?
A. Cervical fracture above C5
B. Cricoid fracture
C. Flail chest
D. Pneumothorax
A

Answer: D
Rationale:
All of the above can cause respiratory distress. Blunt trauma to the airway is most commonly secondary
to direct blows. A passenger in the front seat with only a lap belt is susceptible to hitting his symphysis or neck on the dashboard. Between 10% to 50% of the patients sustaining blunt airway trauma have a cervical spine injury. Respiratory complications are common with cervical spine injuries. The extent of the respiratory derangement is associated with the level of the injury to the cervical spine. While the patient will have some respiratory compromise, diaphragmatic paralysis is spared with injuries at C5 or below. Fractures of the cricoid cartilage are not common. When they do occur there is a 25% incidence of damage to the recurrent laryngeal nerve, which results in vocal cord paralysis and airway compromise. Mortality associated with cricoid fractures is reported to exceed 43%. Application of cricoid pressure to a patient with a cricoid fracture can result in airway obstruction. A flail chest is by definition fractures of three adjacent ribs and results in paradoxical chest wall movement. A pneumothorax may impair respirations but in most situations will not result in acute respiratory distress. This should be distinguished from a tension pneumothorax and an open pneumothorax which can cause acute distress.

122
Q

Which of the following is a property of metoclopramide?
A. Delays gastric emptying
B. Intensifies activity of the vomiting center
C. Increases gastroesophageal sphincter tone
D. Attenuates extrapyramidal effects

A

Answer: C
Rationale:
The incidence of aspiration is relatively low at 5 cases per 10,000. This incidence, however, is markedly increased in the traumatized patient. Pharmacologic measures may decrease the risk of aspiration. Metoclopramide stimulates gastric emptying, attenuates activity of the vomiting center and increases gastroesophageal sphincter tone. It must be administered at least 20 minutes prior to induction and its effect is decreased if administered in conjunction with an opioid. It acts on the dopamine receptor in the chemoreceptor trigger zone and thus can cause extrapyramidal effects. These effects can be treated with benzotropine or diphenhydramine.

123
Q

A 27-year-old individual has suffered from acute head trauma. He has been noted to be urinating 2 to 3 Liters per four hours of light yellow urine over 4 hours with only a small presence of blood noted. A preoperative ECG demonstrates flattened T- waves on his chest leads. Prior to taking this individual to the operating room to repair his fractured mandible, you should consider:
A. obtaining a cardiac work up.
B. obtaining a renal arteriogram.
C. replacing potassium losses.
D. replacing potassium and magnesium losses.

A

Answer D

Rationale:
This individual is likely experiencing diabetes insipidus secondary to head trauma. The flattened T- waves are consistent with hypokalemia, however in order to correct his potassium level he should have his magnesium level adjusted as well (with replacement using 1g of MgSO4).

124
Q
A 60-year-old man is undergoing intravenous general anesthesia for full mouth extractions when he shows signs of labored breathing and cyanosis. The ECG shows evidence of wide complex tachycardia at a rate of 160. The pulse oximeter is registering poor pulse signal. The systolic blood pressure is 65 mm HG. Initial treatment should include:
A. administering amiodarone.
B. administering epinephrine.
C. administering vasopressin.
D. defibrillation.
A

Answer: D
Rationale:
The patient is presenting with an unstable ventricular tachycardia which is treated with prompt defibrillation.

125
Q
A 4 year-old child is seen for extraction of a several carious teeth. Her medical history reveals a repaired ventricular septal defect. On exam, you note a bifid uvula and hypernasal, poorly intelligible speech. She has normal vision and the remainder of her exam is otherwise unremarkable. Which of the following is the most likely diagnosis?
A. Van der Woude syndrome
B. Stickler syndrome
C. Nager syndrome
D. Velocardiofacial syndrome
A

Answer: D
Rationale:
Velocardiofacial syndrome was first described by Shprintzen and colleagues in 1978 as a syndrome comprising clefts of the palate either overt or submucous, congenital heart defects, typical facies and learning difficulties. The hypernasal speech and bifid uvula are suggestive of a submucous cleft palate, which is a feature of the syndrome.
Van der Woude syndrome displays an autosomal dominant pattern of inheritance and consists of lower lip pits and clefts of the lip and/or palate.
Stickler syndrome, also known as Hereditary Arthro-Ophthalmopathy, consists of a flat facies with a short nose, epicanthal folds and anteverted nares, a cleft palate, spondyloepiphyseal dysplasia and high grade myopia.
Nager syndrome is characterized by radial limb hypoplasia, malar hypoplasia with down- slanting palpebral fissures, ear defects and occasionally a cleft palate
Reference:
Smith’s Patterns of Recognizable Human Malformation, ed. 6. Jones ed. Philadelphia: Saunders 2005:976pp.

126
Q
A 60 year-old post-menopausal female presents to the emergency department with the complaint of transient but sudden, severe unilateral headache. Her past medical history is significant for hypertension, tobacco and alcohol abuse. Her neurological examination is normal. This clinical history is most compatible with which diagnosis.
A. variant migraine.
B. cerebral vasospasm.
C. giant cell arteritis.
D. subarachnoid hemorrhage.
A

Answer: D
Rationale:
A retrospective analysis of patients with subarachnoid hemorrhage (SAH) suggests that minor episodes with sudden headache may precede rupture of an aneurysm. Up to 40% of patient will experience a sentinel headache a week or more prior to rupture of a cerebral aneurysm. This woman’s history is compatible with sentinel headache for a “leaking” aneurysm. Risk factors for the development of SAH are 1) smoking, 2)hypertension, and 3)alcohol, 4)estrogen depletion. Variant migraine includes such findings as hemiplegia, brain stem symptoms (vertigo, etc.), transient blindness, ophthalmoplegia, not present in this woman. Giant cell arteritis is associated with unilateral temporal headache, jaw claudication, and possible symptoms of retinal ischemia, not manifested in this woman. Cerebral vasospasm is a complication of SAH leading to neurologic decline in the days following the SAH.
Reference:
Ingall, T, Asplund, K, Mahonen, M, Bonita, R. A multinational comparison of subarachnoid hemorrhage epidemiology in the WHO MONICA stroke study. Stroke 2000; 31:1054
Edlow, JA, Caplan, LR. Avoiding pitfalls in the diagnosis of subarachnoid hemorrhage. N Engl J Med 2000; 342:29
Lipton, RB, Bigal, ME, Steiner, TJ, et al. Classification of primary headaches. Neurology 2004; 63:427
Hunder, GG. Giant cell arteritis and polymyalgia rheumatica. In: Textbook of Rheumatology, 6th ed, Kelly, WN, Harris, ED, Ruddy, S, et al (Eds), WB Saunders Company, Philadelphia, 2001

127
Q

A 38 year-old female with a long history of periodic migraine headache has, for the past 3 months, experienced an insidious increase in headache frequency to near daily headache and frequent awakening with headache. For the past 4 months she has tried to manage her headache with daily over-the-counter acetaminophen, aspirin, and caffeine containing analgesics. Her present condition is best described by:
A. variant migraine.
B. rebound-type transformed migraine. C. cluster headache.
D. hemiplegic migraine.

A

Answer: B
Rationale:
Variant migraine refers anyone of several sub-classifications of migraine such as retinal migraine, hemiplegic migraine, etc. Cluster headache is a distinct clinical syndrome affecting men more than woman, with attacks accompanied by ipsilateral lacrimation, nasal discharge, ptosis, conjunctival injection, and pupillary change. Hemiplegic migraine is a migraine variant characterized by motor and sensory symptoms that are unilateral. Symptoms can last longer than the headache itself and complete recovery may take weeks in some cases. Transformed headache is a transformation from intermittent migraine headache attacks to daily or almost daily headache. The excessive and frequent use of symptomatic medications including ergot derivatives, analgesics and high-dose NSAID’s can result in the insidious transformation to daily headaches. This has been also referred to as medication-overuse headache where daily headache is induced and maintained by the medications used to relieve such pain.
Reference:
Dodick DW. Chronic daily headache. N Engl J Med 2006;354:158-65 Saper JR. Headache disorders. Med Clin North Am 1999; 83:663-90
Corbett, JJ. Neuro-ophthalmic complications of migraine and cluster headaches. Neurol Clin 1983; 1:973